Кинематика 9 класс физика формулы: Ошибка: 404 Материал не найден

Содержание

Равноускоренное движение, вектор ускорения, направление, перемещение. Формулы, определение, законы

Тестирование онлайн

Равноускоренное движение

В этой теме мы рассмотрим очень особенный вид неравномерного движения. Исходя из противопоставления равномерному движению, неравномерное движение — это движение с неодинаковой скоростью, по любой траектории. В чем особенность равноускоренного движения? Это неравномерное движение, но которое «равно ускоряется». Ускорение у нас ассоциируется с увеличением скорости. Вспомним про слово «равно», получим равное увеличение скорости. А как понимать «равное увеличение скорости», как оценить скорость равно увеличивается или нет? Для этого нам потребуется засечь время, оценить скорость через один и тот же интервал времени. Например, машина начинает двигаться, за первые две секунды она развивает скорость до 10 м/с, за следующие две секунды 20 м/с, еще через две секунды она уже двигается со скоростью 30 м/с. Каждые две секунды скорость увеличивается и каждый раз на 10 м/с. Это и есть равноускоренное движение.

Физическая величина, характеризующая то, на сколько каждый раз увеличивается скорость называется ускорением.

Можно ли движение велосипедиста считать равноускоренным, если после остановки в первую минуту его скорость 7км/ч, во вторую — 9км/ч, в третью 12км/ч? Нельзя! Велосипедист ускоряется, но не одинаково, сначала ускорился на 7км/ч (7-0), потом на 2 км/ч (9-7), затем на 3 км/ч (12-9).

Обычно движение с возрастающей по модулю скоростью называют ускоренным движением. Движение же с убывающей скоростью — замедленным движением. Но физики любое движение с изменяющейся скоростью называют ускоренным движением. Трогается ли автомобиль с места (скорость растет!), или тормозит (скорость уменьшается!), в любом случае он движется с ускорением.

Равноускоренное движение — это такое движение тела, при котором его скорость за любые равные промежутки времени изменяется (может увеличиваться или уменьшаться) одинаково

Ускорение тела

Ускорение характеризует быстроту изменения скорости. Это число, на которое изменяется скорость за каждую секунду. Если ускорение тела по модулю велико, это значит, что тело быстро набирает скорость (когда оно разгоняется) или быстро теряет ее (при торможении). Ускорение — это физическая векторная величина, численно равная отношению изменения скорости к промежутку времени, в течение которого это изменение произошло.

Определим ускорение в следующей задаче. В начальный момент времени скорость теплохода была 3 м/с, в конце первой секунды скорость теплохода стала 5 м/с, в конце второй — 7м/с, в конце третьей 9 м/с и т.д. Очевидно, . Но как мы определили? Мы рассматриваем разницу скоростей за одну секунду. В первую секунду 5-3=2, во вторую секунду 7-5=2, в третью 9-7=2. А как быть, если скорости даны не за каждую секунду? Такая задача: начальная скорость теплохода 3 м/с, в конце второй секунды — 7 м/с, в конце четвертой 11 м/с.В этом случае необходимо 11-7= 4, затем 4/2=2. Разницу скоростей мы делим на промежуток времени.

Эту формулу чаще всего при решении задач применяют в видоизмененном виде:

Формула записана не в векторном виде, поэтому знак «+» пишем, когда тело ускоряется, знак «-» — когда замедляется.

Направление вектора ускорения

Направление вектора ускорения изображено на рисунках

На этом рисунке машина движется в положительном направлении вдоль оси Ox, вектор скорости всегда совпадает с направлением движения (направлен вправо). Когда вектор ускорение совпадает с направлением скорости, это означает, что машина разгоняется. Ускорение положительное.

При разгоне направление ускорения совпадает с направлением скорости. Ускорение положительное.

На этом рисунке машина движется в положительном направлении по оси Ox, вектор скорости совпадает с направлением движения (направлен вправо), ускорение НЕ совпадает с направлением скорости, это означает, что машина тормозит. Ускорение отрицательное.

При торможении направление ускорения противоположно направлению скорости. Ускорение отрицательное.

Разберемся, почему при торможении ускорение отрицательное. Например, теплоход за первую секунду сбросил скорость с 9м/с до 7м/с, за вторую секунду до 5м/с, за третью до 3м/с. Скорость изменяется на «-2м/с». 3-5=-2; 5-7=-2; 7-9=-2м/с. Вот откуда появляется отрицательное значение ускорения.

При решении задач, если тело замедляется, ускорение в формулы подставляется со знаком «минус»!!!

Перемещение при равноускоренном движении

Дополнительная формула, которую называют безвременной

Формула в координатах

Связь со средней скоростью

При равноускоренном движении среднюю скорость можно рассчитывать как
среднеарифметическое начальной и конечной скорости

Из этого правила следует формула, которую очень удобно использовать при решении многих задач

Соотношение путей

Если тело движется равноускоренно, начальная скорость нулевая, то пути, проходимые в последовательные равные промежутки времени, относятся как последовательный ряд нечетных чисел.

Главное запомнить

1) Что такое равноускоренное движение;
2) Что характеризует ускорение;
3) Ускорение — вектор. Если тело разгоняется ускорение положительное, если замедляется — ускорение отрицательное;
3) Направление вектора ускорения;
4) Формулы, единицы измерения в СИ

Упражнения

Два поезда идут навстречу друг другу: один — ускоренно на север, другой — замедленно на юг. Как направлены ускорения поездов?

Одинаково на север. Потому что у первого поезда ускорение совпадает по направлению с движением, а у второго — противоположное движению (он замедляется).

Поезд движется равноускоренно с ускорением a (a>0). Известно, что к концу четвертой секунды скорость поезда равна 6м/с. Что можно сказать о величине пути, пройденном за четвертую секунду? Будет ли этот путь больше, меньше или равен 6м?

Так как поезд движется с ускорением, то скорость его все время возрастает (a>0). Если к концу четвертой секунды скорость равна 6м/с, то в начале четвертой секунды она была меньше 6м/с. Следовательно, путь, пройденный поездом за четвертую секунду, меньше 6м.

Какие из приведенных зависимостей описывают равноускоренное движение?

Уравнение скорости движущегося тела . Каково соответствующее уравнение пути?

*Автомобиль прошел за первую секунду 1м, за вторую секунду 2м, за третью секунду 3м, за четвертую секунду 4м и т.д. Можно ли считать такое движение равноускоренным?

В равноускоренном движении пути, проходимые в последовательные равные промежутки времени, относятся как последовательный ряд нечетных чисел. Следовательно, описанное движение не равноускоренное.

Все ⚠️ формулы по физике за 9 класс: определения, пояснения

Формулы по физике за 9 класс: основные разделы

Программа обучения по предмету физика в 9 классе включает в себя несколько разделов: кинематика и динамика, которые в свою очередь состоят из подразделов. Таким образом ученики старшей школы изучают механические колебания и волны, законы взаимодействия и движения тел, электромагнитные явления, строение атомов и их ядер, основные законы механики. В школьную программу девятого года обучения также входят основные свойства света: интерференция, преломление и дисперсия.  

Кинематика

Кинематика — один из разделов механики. Кинематика изучает механическое движение тел и способы его описания, независимо от причин этого движения. В данном случае под механическим движением подразумевается любое изменение положения какого-либо тела полностью или частично относительно других тел, случившееся с течением времени.

В Кинематике изучают простые виды движения.

Осторожно! Если преподаватель обнаружит плагиат в работе, не избежать крупных проблем (вплоть до отчисления). Если нет возможности написать самому, закажите тут.

Равномерное прямолинейное движение

Понятие равномерного прямолинейного движения заключается в том, что тело движется по прямой с одинаковой скоростью, то есть за равные промежутки времени тело перемещается на одинаковое расстояние. В таком случае скорость тела остается постоянной, однако является векторной величиной.

Скорость может быть как положительной, так и отрицательной. Все зависит от того, в каком направлении оси X (положительном или отрицательно) направлен вектор скорости. Если тело находится в покое, то его скорость равняется нулю, а координата не меняется в течение времени.

При равномерном прямолинейном движении координата тела вычисляется по следующей формуле:

В этой формуле x0 – начальная координата, x – конечная координата, v – скорость, t – время.

Если начальная координата — это начало движения и x0 = 0, то формулу можно сократить до x1 = v · t.

Если x0 = 0, то пройденный путь S будет равен координате x. Из этого утверждения можно получить формулу прямолинейного равномерного движения относительно пройденного телом расстояния:

Из этого можно вывести формулы относительно скорости и времени: 

Скорость и время также можно выразить из полной формулы для тех случаев, когда x0 не равно 0:

v = (x1 — x0)/t и t = (x1 — x0)/v.

Равноускоренное прямолинейное движение

В случае равноускоренного прямолинейного движения тело изменяет скорость своего движения на одинаковую величину за любые равные промежутки времени. Под ускорением в контексте данного определения понимается изменение значения скорости за единицу времени.

Скорость тела вычисляется по формуле:

В данной формуле v – конечная скорость, v0 – начальная скорость, a – ускорение, t – время.

В равноускоренном прямолинейном движении постоянной величиной является ускорение, а не скорость. Ускорение может быть больше или меньше нуля. В случае увеличения скорость, значение ускорения будет больше нуля, а в случае уменьшения — меньше. 

Рассмотрим случай, если начальная скорость тела равно 0. Тогда его скорость через какое-либо время t будет равна произведению ускорения и этого времени:

Допустим, что нам известны текущая скорость тела и время, за которое тело развило указанную скорость из состояния покоя. Тогда мы можем определить ускорение:

В том случае, если начальная скорость тела не равна нулю, мы можем рассчитать конечную скорость тела по следующей формуле:

Взглянем на случаи, когда вектор скорости направлен в противоположном направлении (например, подброшенный камень, его скорость направлена в противоположную сторону от ускорения свободного падения) или в случае торможения. Тогда формула будет выглядеть следующим образом:

В случае свободного ускорения остальные формулы будут записываться так:

at = v – v0, a = (v – v)/t

А говоря о торможении, мы используем эти формулы:

at = v0 – v, a = (v0 – v)/t

Если тело останавливается, то нам следует использовать эту формулу:

А если необходимо узнать, через какой отрезок времени тело остановится, то мы запишем формулу так:

Обратимся к формуле, которая поможет найти путь, которое тело проходит при прямолинейном ускорении. Если при равномерном движении, оси времени и расстояния параллельны, то в случае равноускоренного движения ось движения либо возрастает, либо убывает. Тогда вместо прямоугольника, чью площадь мы вычисляли при равномерном движении, необходимо вычислить площадь трапеции.

Площадь трапеции равна полусумме оснований на высоту, таким образом мы получаем:

Пройденный путь определяется по формуле:

Путь торможения рассчитывается с помощью этой формулы:

Равномерное движение по окружности

Говоря о равномерном движении по окружности, нужно понимать, что в этом случае вектор скорости тела изменяется (скорость направлена по касательным к окружности), а модуль скорости тела (числовое значение) остается постоянным.

Предположим, что необходимо вычислить модуль скорости за один оборот тела по окружности. Обозначим оборот как S, а время, за которое тело его совершило, как t. Тогда формула будет записываться следующим образом: v = s/t.

Однако, если мы говорим об одном обороте, то это называется период. То есть время, за которое тело совершает один оборот вокруг окружности. Он обозначается как T. И тогда формула одного оборота будет выглядеть так: v = s/T

Если S в данном случае это длина окружности (l), то формула принимает вид v = 2πR/T, в соответствии с формулой окружности l = 2\piR

Если необходимо найти период при известном модуле скорости, то формула примет вид T = 2\piR/v

Аналогично радиус можно найти через формулу R = ½ vT/\pi

Динамика

Динамика — раздел механики, изучающий предпосылки изменения в характере движения. Например, возникновение движения. Именно этот раздел изучает три закона Ньютона. В задачах динамики содержится решение таких вопросов как определение действующих на тело сил по характеру его движения и наоборот.

Законы Ньютона

Первый закон Ньютона гласит, что существуют такие системы отсчета, относительно которых тело движется прямолинейно и равномерно, или покоится, если на него не действуют другие тела или их действия скомпенсированы.

Введем основные величины:

Инерциальными называются системы отсчета, которые движутся равномерно прямолинейно относительно Земли. Все системы отсчета, которые движутся прямолинейно и равномерно относительно инерциальной, также являются таковыми. Если система отсчета движется с ускорением, то она — неинерциальная.

Сила — это физическая величина, которая характеризует действие одного тела на второе. В результате этого действия второе тело получает ускорение в инерциальной системе отсчета. Измеряется в ньютонах.

Масса — это физическая величина, которая количественно характеризует инертность тела. Измеряется в килограммах.

Взглянем на тело, на которое действует сила с модулем равным 1 Н. Так как изначально тело массой 1 кг находилось в покое в инерциальной системе, модуль его ускорения будет равен 1 м/с2.  

В соответствии со вторым законом Ньютона сила, действующая на тело, равна произведению массы тела на сообщаемое этой силой ускорение. Это основной закон динамики.

Для выведения второго закона Ньютона и формулы, где F = ma, необходимо обобщить два факта:

  • если на два тела, масса которых различна, подействовать равной силой, то ускорения, которые приобретут тела, будут обратно пропорциональны массам;
  • если на одно и то же тело действуют силы разной величины, то ускорения тела будут прямо пропорциональны приложенным силам.

Благодаря этому закону, возможно вычислить не только силу, действующую на тело, но и ускорение. Для этого нужно использовать формулу \[w = \frac{F}{m}\]

В векторной форме второй закон Ньютона записывается как ma = mg + N + Fтр

Третий закон Ньютона гласит, что силы, с которыми две материальные точки воздействуют друг на друга, всегда равны по модулю и направлены в противоположные стороны вдоль прямой, соединяющей эти точки.

Выразить закон формулой можно следующим образом F1 = -F2

Примечание

В случае взаимодействия тел силы имеют одинаковую природу, однако, они приложены к разным телам. Таким образом эти силы не могут уравновешивать друг друга, а складывать можно только силы, приложенные к одному телу.

Силы в природе

В соответствии с законом Гука, при деформации тела возникает сила, которая стремится восстановить прежние размеры и форму тела. Природа этой силы состоит в электромагнитном взаимодействии между атомами и молекулами вещества. Эта сила называется упругость.

Говоря о малых деформациях (если |x| < l) сила упругости пропорциональна деформации тела и направлена в противоположную сторону от перемещения частиц тела при деформации. Исходя из этого формула выглядит следующим образом Fx = Fупр = -kx

В данном случае коэффициент k — жесткость тела, она измеряется в ньютонах на метр (Н/м).

Физике свойственен другой способ записи закона Гука. В его записи используются понятия относительной деформации и напряжения. Относительная деформация ε = x / l, а напряжение — это отношение силы к площади поперечного сечения деформированного тела δ = F / S = -Fупр / S.

Исходя из этого, закон Гука можно сформулировать так \[ε = \frac{E}{δ}\]

Коэффициент Е — это модуль Юнга. Он зависит исключительно от свойств материала. Размеры и форма не имеют значения.  

Если говорить о случаях сложных деформациях, например в случае деформации изгиба, в формуле появляется сила N — сила реакции опоры. Эта сила направлена перпендикулярно поверхности соприкосновения. N = -mg

Сила всемирного тяготения

Закон всемирного тяготения говорит, что все тела притягиваются друг к другу с силами, прямо пропорциональными их массам и обратно пропорциональна квадрату расстояния между ними.

Эту силу можно вычислить по формуле F = G\fracMR32m = mg

g в данном случае — ускорение свободного падения, о котором говорилось выше. В данном случае g = G\fracMR32. Среднее значение ускорения свободного падения равно 9,81 м/с2

R3 — это радиус Земли. Он равен 6,38·106 м.

G в формуле обозначает гравитационную постоянную. Она равна 6,67·10–11 Н·м2/кг2.

Движение тела под действием силы тяжести

Ускорение свободного падение является частным случаем равноускоренного прямолинейного движения. В этом случае ускорение всегда будет равно 9,8 м/с2 и обозначается буквой g. Таким образом g — это ускорение свободного падения.

Ускорение свободного падения можно вычислить по следующей формуле: g = GM3/(R3+H)2

В данном случае H — это гравитационная постоянная, M — масса земли, R — радиус земли, а H — высота падения тела.

Скорость тела под действием силы тяжести можно вычислить по формуле: v = gt

Высоту, с которой падает тело, можно вычислить по формуле H=gt2/2

Силы трения

Силой трения называют силу, характеризующую взаимодействие, возникающее в месте соприкосновения тел и препятствующее их относительному движению. Сила трения имеет электромагнитную природу.

Трение можно разделить на три вида: трение покоя, трение скольжения и трение качения.

Трение покоя — это трение, которое возникает при отсутствии перемещения соприкасающихся тел относительно друг друга.

Можно сказать, что эта сила не позволяет одному телу двигаться относительно другого. Эта сила направлена противоположно силе, приложенной извне параллельно поверхности соприкосновения. Сила трения покоя возрастает вместе с силой, которая стремится сдвинуть тело с места.

Трение скольжения возникает при действии на тело силе, превышающей максимальную силу трения покоя.

Это тело сдвигается с места и начинает дальнейшее движение. Сила трения скольжения всегда направлена в противоположную сторону от относительной скорости соприкасающихся тел.

Трение качения возникает в случае, если тело не скользит по другому телу, а катится наподобие колеса или цилиндра. Трение качения — это трение, которое возникает на месте их соприкосновения.

В виде формулы сила трения выглядит следующим образом: Fтр = μmg

В данном случае μ – коэффициент трения, m – масса тела, а g — ускорение свободного падения (постоянная величина 9,81 м/с2).

Движение тела под действием нескольких сил

Если на тело действуют несколько сил одновременно, то необходимо найти равнодействующую всех сил по формуле F = F1 + F2 + F3

Равнодействующая сила может быть равна нулю. В таком случае тело находится в состоянии покоя.

Закон сохранения в механике

Закон сохранения импульса гласит, что геометрическая сумма импульсов тел, составляющих замкнутую систему, остается постоянной при любых движениях и взаимодействиях тел системы.

В виде формулы закон сохранения импульса выглядит следующим образом: p1 + p2 = p1’ + p2’ m1v + m2u = m1v’ + m2u’

В свою очередь импульсом тела называют величину, которая равна произведению массы тела на его скорость: p = mv.

Изменение импульса тела равно импульсу силы, который в свою очередь вычисляется по формуле P = Ft

Кинетическая энергия равна половине произведения массы тела и квадрата его скорости: Ek = mv2/2

Кинетическая энергия — это физическая величина, которая характеризует движущиеся тела. Выражается в Дж.

Закон сохранения энергии состоит в том, что полная механическая энергия замкнутой системы тел, взаимодействующих силами тяготения или силами упругости, остается неизменной при любых движениях тел системы. 

Мощность — это величина, которая равна отношению совершенной работы к промежутку времени, за который она совершена. Выражается в Вт.

Вычисляется по формуле N = A/t

Коэффициент полезного действия (КПД) — это название величины, равной отношению полезной работы ко всей совершенной работе. Выражается в Дж.

КПД демонстрирует эффективность использования затраченной энергии. Коэффициент не может быть больше единицы, однако его можно выразить в процентах.2}{2}\]\)

Примеры задач

Задача №1

На рисунке представлены графики зависимости координаты двух тел от времени. Графики каких зависимостей показаны? Какой вид имеют графики зависимости скорости и пути пройденного телом, от времени?

Решение

На рисунке показаны графики равномерного движения тел.

  1. В начальный момент времени t = 0 первое тело имеет начальную координату хо1 = 1 м, второе тело — координату хо2 = 0.
  2. Оба тела движутся в направлении оси Х, так как координата возрастает с течением времени.
  3. Уравнение движения для равномерного прямолинейного движения имеет вид: x=xо+vхt.

Тогда для первого, второго тела соответственно:

x1=xо1+vt   и   x2=xо2+vt

или x1=1+vt и  x2=vt.

Определим скорости первого и второго тела:

\(\[v_{1x} = \frac{x_{1} — 1}{t} = \frac{2 — 1}{2} = 0,5 м\с\]\)

\(\[v_{2x} = \frac{x_{2}}{t} = \frac{1}{2} = 0,5 м\с\]\)


Задача №2

Шар подвешен на невесомой нерастяжимой нити длиной l = 0,5 м.2}{2} = 2mgl + \frac{mgl}{2}\]\)

vo2 = g4l + gl = 5gl

vo = √(5gl)

Выполнив вычисления, получим: vo = √(5×10×0,5) = 5 (м/с).

Ответ: если шарик подвешен на нерастяжимой нити, его скорость должна составлять не менее 5 м/с.

Задача №3

Экваториальный радиус Земли равен 6370 км. Определить линейную и угловую скорости движения точек экватора при вращении Земли вокруг оси.

Решение:

Линейная скорость вращения ν точек земного экватора:

\(\[v = \frac{2\piR}{T}\]\)

При этом угловая скорость вращения w всех точек Земли равна:

\(\[w = \frac{2\pi}{T}\]\)

После вычислений у нас получится: ν = 463 м/с, w = 7,3×10−5 рад/с.

9 класс Формулы Формула Название Обозначение и единицы измерения в СИ Путь при равномерном прямолинейном движении

Образовательный минимум

триместр предмет физика класс 9т Образовательный минимум Основные понятия Движения тела по вертикали, брошенного под углом к горизонту, горизонтально. Движение по с постоянной по модулю скоростью. Центростремительное

Подробнее

Рабочая программа по физике 9 класс

Государственное бюджетное общеобразовательное учреждение Республики Хакасия «Хакасская национальная гимназия-интернат им. Н.Ф.Катанова» «СОГЛАСОВАНО» УТВЕРЖДЕНО на заседании МО естественно — математических

Подробнее

СОДЕРЖАНИЕ ПРОГРАММЫ I.

Данная программа учебного предмета «Физика» для обучающихся 9 класса разработана на основе требований к результатам освоения ООП ООО МКОУ «Большеокинская СОШ» в соответствии с ФК ГОС ООО (2004 г.). Планируемые

Подробнее

Пояснительная записка

Пояснительная записка Программа составлена в соответствии с Федеральным компонентом государственного стандарта основного общего образования по физике (Приказ Минобразования России от 05.03.2004 089 «Об

Подробнее

Пояснительная записка

Пояснительная записка Данная программа учебного предмета «Физика» для обучающихся 9 класса муниципального казённого общеобразовательного учреждения «Большеокинская СОШ» разработана на основе авторской

Подробнее

Требования к уровню подготовки учащихся

Требования к уровню подготовки учащихся В результате изучения физики ученик должен знать/понимать: смысл понятий: физическое явление, физический закон, взаимодействие, электрическое поле, магнитное поле,

Подробнее

Требования к уровню подготовки учащихся

Ученик должен знать/понимать: уметь Требования к уровню подготовки учащихся смысл понятий: физическое явление, физический закон, взаимодействие, электрическое поле, магнитное поле, волна, атом, атомное

Подробнее

НАУЧНО-ОБРАЗОВАТЕЛЬНЫЙ МАТЕРИАЛ

НЕКОММЕРЧЕСКАЯ ОРГАНИЗАЦИЯ «АССОЦИАЦИЯ МОСКОВСКИХ ВУЗОВ» ГОСУДАРСТВЕННОЕ ОБРАЗОВАТЕЛЬНОЕ УЧРЕЖДЕНИЕ ВЫСШЕГО ПРОФЕССИОНАЛЬНОГО ОБРАЗОВАНИЯ МОСКОВСКИЙ ГОСУДАРСТВЕННЫЙ УНИВЕРСИТЕТ ГЕОДЕЗИИ И КАРТОГРАФИИ НАУЧНО-ОБРАЗОВАТЕЛЬНЫЙ

Подробнее

ФИЗИКА. Темы вступительного собеседования

ФИЗИКА Темы вступительного собеседования для поступления в 9 класс Механическое движение (равномерное и неравномерное) и его основные характеристики (путь, перемещение, траектория, скорость, величин. Взаимодействие

Подробнее

Рабочая программа по физике в 9 классе

Муниципальное общеобразовательное бюджетное учреждение «Восточненская средняя общеобразовательная школа» Тындинского района Рабочая программа по физике в 9 классе Учитель: Константинова Галина Дмитриевна

Подробнее

УЧЕБНО-ТЕМАТИЧЕСКИЙ ПЛАН

УЧЕБНО-ТЕМАТИЧЕСКИЙ ПЛАН Название Колво Изучение Кол- Кол- раз. нового и во во час. закрепление к/р л/р 1 Законы взаимодействия и движения тел 43 39 2 2 2 Механические колебания и волны. Звук. 12 10 1

Подробнее

учебный год

Приложение к рабочей программе по физике для 9 класса Примерные оценочные и методические материалы для осуществления текущего контроля успеваемости и промежуточной аттестации учащихся 9-го класса по физике

Подробнее

Структура и содержание. Паспорт

Спецификация оценочных материалов для проведения тематических контрольных работ по физике в 9 классе Структура и содержание Паспорт Раздел, тема Проверяемые компетенции (из рабочей программы) Вид оценочных

Подробнее

Пояснительная записка

2 Пояснительная записка Рабочая программа составлена на основе программы «Физика, 9 класс, Грачев А.В., Погожев В.А., Селиверстов А.В., 2011 г. — В кн. Грачев А.В., Погожев В.А., Селиверстов А.В. Физика:

Подробнее

Пояснительная записка

Пояснительная записка Рабочая программа составлена на основе регионального базисного учебного плана для общеобразовательных учебных учреждений Курской области. Основой для составления программы по физике

Подробнее

Powered by TCPDF (www.tcpdf.org)

Powered by TCPDF (www.tcpdf.org) Пояснительная записка. Статус программы Данная рабочая программа по физике составлена на основе «Программы основного общего образования. Физика. 7-9 классы», авторы: А.

Подробнее

ПОЯСНИТЕЛЬНАЯ ЗАПИСКА

ПОЯСНИТЕЛЬНАЯ ЗАПИСКА Тематическое планирование составлено на основе Федерального компонента государственного стандарта среднего (полного) общего образования в соответствии с пособием «Программы основного

Подробнее

«Школа 32» г.н.новгород

Муниципальное бюджетное общеобразовательное учреждение «Школа 32» г.н.новгород РАБОЧАЯ ПРОГРАММА ПЕДАГОГА по физике 9 класса 2016 2017 учебный год Рабочая программа для 9 класса составлена в соответствии

Подробнее

Планируемые результаты изучения физики в 9 классе: Выпускник научится использовать термины: физическое явление, физический закон, вещество,

Планируемые результаты изучения физики в 9 классе: Выпускник научится использовать термины: физическое явление, физический закон, вещество, взаимодействие, электрическое поле, магнитное поле, волна, атом,

Подробнее

Пояснительная записка

Пояснительная записка Физика как наука о наиболее общих законах природы, выступая в качестве учебного предмета в школе, вносит существенный вклад в систему знаний об окружающем мире. Она раскрывает роль

Подробнее

РАБОЧАЯ ПРОГРАММА. по физике. 9 класс

Муниципальное бюджетное общеобразовательное учреждение «Школа 20» «Рассмотрено» на заседании школьного методического объединения Председатель: Борисова А.В. Протокол 30.08.207г. «Согласовано» заместитель

Подробнее

«МЕХАНИКА 1» Величины в формуле

Величина, её определение Обозначение Единица измерения «МЕХАНИКА 1» Формла Величины в формле ВИДЫ ДВИЖЕНИЯ I. Равномерное прямолинейное движение-это движение, при котором тело за любые равные промежтки

Подробнее

Все формулы по физике 9 класса

ЗАКОНЫ ВЗАИМОДЕЙСТВИЯ И ДВИЖЕНИЯ ТЕЛ
Вычисление перемещения АВ2 = АС2 + ВС2Перемещение – вектор, соединяющий начальную точку движения тела с его конечной точкой.
Проекция вектора перемещенияSx = x2 – x1x1 – начальная координата, [м]
x2 – конечная координата, [м]
Sx – перемещение, [м]
Формула расчета скорости движения телаv = s/tСкорость – физическая величина, равная отношению перемещения к промежутку времени, за которое это перемещение произошло.v – скорость, [м/с]
s – путь, [м]
t – время, [c]
Уравнение движенияx = x0 + vxtx0 – начальная координата, [м]
x – конечная координата, [м]
v – скорость, [м/с]
t – время, [c]
Формула для вычисления ускорения движения телаa = v — v0⃗/tУскорение – физическая величина, которая характеризует быстроту изменения скорости.a – ускорение, [м/с2]
v – конечная скорость, [м/с]
v0 – начальная скорость, [м/с]
t – время, [c]
Уравнение скоростиv = v0⃗+ atv – конечная скорость, [м/с]
v0 – начальная скорость, [м/с]
a – ускорение, [м/с2]
t – время, [c]
Уравнение ГалилеяS = v0t + at2/2S – перемещение, [м]
v – конечная скорость, [м/с]
v0 – начальная скорость, [м/с]
a – ускорение, [м/с2]
t – время, [c]
Закон изменения координаты тела при прямолинейном равноускоренном движенииx = x0 + v0t + at2/2x0 – начальная координата, [м]
x – конечная координата, [м]
v – конечная скорость, [м/с]
v0 – начальная скорость, [м/с]
a – ускорение, [м/с2]
t – время, [c]
Первый закон НьютонаЕсли на тело не действуют никакие тела либо их действие скомпенсировано, то это тело будет находиться в состоянии покоя или двигаться равномерно и прямолинейно.
Второй закон Ньютонаa = F ⃗/mУскорение, приобретаемое телом под действием силы, прямо пропорционально величине этой силы и обратно пропорционально массе тела.a – ускорение, [м/с2]
F – сила, [Н]
m – масса, [кг]
Третий закон Ньютона|F1⃗ |=|F2⃗|
F11 ⃗ = -F2
Сила, с которой первое тело действует на второе, равна по модулю и противоположна по направлению силе, с которой второе тело действует на первоеF – сила, [Н]
Формула для вычисления высоты, с которой падает телоH=gt2/2Н – высота, [м]
t – время, [c]
g ≈ 9,81 м/с2 – ускорение свободного падения
Формула для вычисления высоты при движении вертикально вверхh=v0t — gt2/2h – высота, [м]
v0 – начальная скорость, [м/с]
t – время, [c]
g ≈ 9,81 м/с2 – ускорение свободного падения
Формула для вычисления веса тела при движении вверх с ускорениемP = m (g + a)P – вес тела, [Н]
m – масса тела, [кг]
g ≈ 9,81 м/с2 – ускорение свободного падения
a – ускорение тела, [м/с2]
Формула для вычисления веса тела при движении вниз с ускорениемP = m (g – a)P – вес тела, [Н]
m – масса тела, [кг]
g ≈ 9,81 м/с2 – ускорение свободного падения
a – ускорение тела, [м/с2]
Формула законаF = Gm1m2/r2Закон всемирного тяготения: два тела притягиваются друг к другу с силой, прямо пропорциональной произведению масс этих тел и обратно пропорциональной квадрату расстояния между ними.F – сила, [Н]
G = 6,67 · 10-11 [Н·м2/кг2] – гравитационная постоянная
m – масса тела, [кг]
r – расстояние между телами, [м]
Формула расчета ускорения свободного падения на разных планетахg = G Mпл/Rпл2g – ускорение свободного падения, [м/с2]
G = 6,67 · 10-11 [Н·м2/кг2 – гравитационная постоянная
M – масса планеты, [кг]
R – радиус планеты, [м]
Формула расчета ускорения свободного паденияg = GM3/(R3+H)2g – ускорение свободного падения, [м/с2]
G = 6,67 · 10-11 [Н·м2/кг2 – гравитационная постоянная
M – масса Земли, [кг]
R – радиус Земли, [м]
Н – высота тела над Землей, [м]
Формула расчета центростремительного ускоренияа=υ2/ra – центростремительное ускорение, [м/с2]
v – скорость, [м/с]
r – радиус окружности, [м]
Формула периода движения по окружностиT = 1/ν = (2πr)/υ = t/NТ – период, [с]
ν – частота вращения,
-1]
t – время, [с]
N – число оборотов
Формула расчета угловой скоростиω = 2π/T = 2πν = υrω – угловая скорость, [рад/с]
υ – линейная скорость, [м/с]
Т – период, [с]
ν – частота вращения, [с-1]
r – радиус окружности, [м]
Формула импульса телаp = mvИмпульсом называют произведение массы тела на его скорость.p – импульс тела, [кг·м/с]
m – масса тела, [кг]
υ – скорость, [м/с]
Формула закона сохранения импульсаp1 + p2 = p1’ + p2
m1v + m2u = m1v’ + m2u’
Закон сохранения импульса: в замкнутой системе импульс всех тел остается величиной постоянной.p – импульс тела, [кг·м/с]
m – масса тела, [кг]
υ – скорость 1-го тела, [м/с]
u – скорость 2-го тела, [м/с]
Формула импульса силыP = Ftp – импульс тела, [кг·м/с]
F – сила, [Н]
t – время, [c]
Формула механической работыA = FsМеханическая работа – физическая величина, равная произведению модуля силы на величину перемещения тела в направлении действия силыA – работа, [Дж]
F – сила, [Н]
s – пройденный путь, [м]
Формула расчета мощностиN = A/tМощность – физическая величина, характеризующая быстроту совершения механической работы.N – мощность, [Вт]
A – работа, [Дж]
t – время, [c]
Формула для нахождения коэффициента полезного действия (КПД)η = Aп/Aз∙100КПД – отношение полезной работы к затраченной работе.Aп – полезная работа, [Дж]
Aз – затраченная работа, [Дж]
Формула расчета потенциальной энергииEk = mv2/2Кинетическая энергия – энергия, которой обладает тело вследствие своего движения.Ek – кинетическая энергия тела, [Дж]
m – масса тела, [кг]
v – скорость движения тела, [м/с]
Формула закона сохранения полной механической энергииmv12/2 + mgh1 = mv22/2 + mgh2Закон сохранения полной механической энергии: полная механическая энергия тела, на которое не действуют силы трения и сопротивления, в процессе его движения остается неизменной.m – масса тела, [кг]
g ≈ 9,81 м/с2 – ускорение свободного падения
v1 – скорость тела в начальный момент времени, [м/с]
v2 – скорость тела в конечный момент времени, [м/с]
h1 – начальная высота, [м]
h2 – конечная высота, [м]
Формула силы тренияFтр = μmgСила трения – сила, возникающая при соприкосновении двух тел и препятствующая их относительному движению.Fтр – сила трения, [Н]
μ – коэффициент трения
m – масса тела, [кг]
g ≈ 9,81 м/с2 – ускорение свободного падения
Уравнение колебанийx = A cos (ωt + φ0)А – амплитуда колебаний, [м]
х – смещение, [м]
t – время, [c]
ω – циклическая частота, [рад/с]
φ0 – начальная фаза, [рад]
Формула периодаT = 1/ν = 2πr/υ = t/NТ – период, [с]
ν – частота колебании, [с-1]
t – время колебании, [с]
N – число колебаний
Формула периода для математического маятникаT= 2π √L/gТ – период, [с]
g ≈ 9,81 м/с2 – ускорение свободного падения
L – длина нити, [м]
Формула периода для пружинного маятникаT = 2π √m/KТ – период, [с]
m – масса груза, [кг]
К – жесткость пружины, [Н/м]
Формула длины волныλ = υТ = υ/νλ – длина волны, [м]
Т – период, [с]
ν – частота, [с-1]
υ – скорость волны, [м/с]
Формула расчета плотности телаρ=m/VПлотность вещества – показывает, чему равна масса вещества в единице объема.ρ – плотность, [кг/м3]
m – масса, [кг]
V – объем тела, [м3]
Формула гидростатического давления жидкостиp = ρghp – давление, [Па], [Н/м]
ρ – плотность жидкости, [кг/м3]
g ≈ 9,81 м/с2 – ускорение свободного падения
h – высота столба жидкости, [м]
Формула силы АрхимедаFA = ρgVЗакон Архимеда: на всякое тело, погруженное в жидкость (газ(, действует выталкивающая сила, равная весу вытесненной жидкости (газа).FА – сила Архимеда, [Н]
ρ – плотность жидкости или газа [кг/м3]
g ≈ 9,81 м/с2 – ускорение свободного падения
V – объем тела, [м3]
ЭЛЕКТРОМАГНИТНОЕ ПОЛЕ
Формула расчета силы АмпераFA = BIL sinαЗакон Ампера: сила действия однородного магнитного поля на проводник с током прямо пропорциональна силе тока, длине проводника, модулю вектора индукции магнитного поля, синусу угла между вектором индукции магнитного поля и проводником.FA – сила Ампера, [Н]
В – магнитная индукция, [Тл]
I – сила тока, [А]
L – длина проводника, [м]
Формула расчета силы ЛоренцаFл = q B υ sinαСила Лоренца – сила, действующая на точечную заряженную частицу, движущуюся в магнитном поле. Она равна произведению заряда, модуля скорости частицы, модуля вектора индукции магнитного поля и синуса угла между вектором магнитного поля и скоростью движения частицы.Fл – сила Лоренца, [Н]
q – заряд, [Кл]
В – магнитная индукция, [Тл]
υ – скорость движения заряда, [м/с]
Формула радиуса движения частицы в магнитном полеr = mυ/qBr – радиус окружности, по которой движется частица в магнитном поле, [м]
m – масса частицы, [кг]
q – заряд, [Кл]
В – магнитная индукция, [Тл]
υ – скорость движения заряда, [м/с]
Формула для вычисления магнитного потокаФ = B S cosαФ – магнитный поток, [Вб]
В – магнитная индукция, [Тл]
S – площадь контура, [м2]
Формула для вычисления величины зарядаq = ItЗаряд – это есть произведение силы тока на время, в течение которого этот заряд протекает по проводнику.q – заряд, [Кл]
I – сила тока, [А]
t – время, [c]
Закон Ома для участка цепиI=U/RЗакон Ома: сила тока в участке цепи прямо пропорциональна напряжению на концах этого участка и обратно пропорциональна его сопротивлению.I – сила тока, [А]
U – напряжение, [В]
R – сопротивление, [Ом]
Формула для вычисления удельного сопротивления проводникаR = ρ * L/S
ρ = R * S/L
Удельное сопротивление – величина, характеризующая электрические свойства вещества, из которого изготовлен проводник.ρ – удельное сопротивление вещества, [Ом·мм2/м]
R – сопротивление, [Ом]
S – площадь поперечного сечения проводника, [мм2]
L – длина проводника, [м]
Законы последовательного соединения проводниковI = I1 = I2
U = U1 + U2
Rобщ = R1 + R2
Последовательным соединением называется соединение, когда элементы идут друг за другом.I – сила тока, [А]
U – напряжение, [В]
R – сопротивление, [Ом]
Законы параллельного соединения проводниковU = U1 = U2
I = I1 + I2
1/Rобщ = 1/R1 +1/R2
Параллельным соединением проводников называется такое соединение, при котором начала и концы проводников соединяются вместе.I – сила тока, [А]
U – напряжение, [В]
R – сопротивление, [Ом]
Формула для вычисления величины заряда.q = ItЗаряд – это есть произведение силы тока на время, в течение которого этот заряд протекает по проводнику.q – заряд, [Кл]
I – сила тока, [А]
t – время, [c]
Формула для нахождения работы электрического токаA = Uq
A = UIt
Работа – это величина, которая характеризует превращение энергии из одного вида в другой, т.е. показывает, как энергия электрического тока, будет превращаться в другие виды энергии – механическую, тепловую и т. д.
Работа электрического поля – это произведение электрического напряжения на заряд, протекающий по проводнику. Работа, совершаемая для перемещения электрического заряда в электрическом поле.
A – работа электрического тока, [Дж]
U – напряжение на концах участка, [В]
q – заряд, [Кл]
I – сила тока, [А]
t – время, [c]
Формула электрической мощностиP = A/t
P = UI
P = U2/R
Мощность – работа, выполненная в единицу времени.P – электрическая мощность, [Вт]
A – работа электрического тока, [Дж]
t – время, [c]
U – напряжение на концах участка, [В]
I – сила тока, [А]
R – сопротивление, [Ом]
Формула закона Джоуля-ЛенцаQ = I2RtЗакон Джоуля-Ленца: при прохождении электрического тока по проводнику количество теплоты, выделяемое в проводнике, прямо пропорционально квадрату тока, сопротивлению проводника и времени, в течение которого электрический ток протекал по проводнику.Q – количество теплоты, [Дж]
I – сила тока, [А];
t – время, [с].
R – сопротивление, [Ом].
Закон отражения светаЛуч падающий, луч отраженный и перпендикуляр, восставленный в точку падения луча, лежат в одной плоскости, при этом угол падения луча равен углу отражения луча.
Закон преломленияsinα/sinγ = n2/n1При увеличении угла падения увеличивается и угол преломления, то есть при угле падения, близком к 90°, преломлённый луч практически исчезает, а вся энергия падающего луча переходит в энергию отражённого.n – показатель преломления одного вещества относительно другого
Формула вычисления абсолютного показателя преломления веществаn = c/vАбсолютный показатель преломления вещества – величина, равная отношению скорости света в вакууме к скорости света в данной среде.n – абсолютный показатель преломления вещества
c – скорость света в вакууме, [м/с]
v – скорость света в данной среде, [м/с]
Закон Снеллиусаsinα/sinγ = v1/v2 = nЗакон Снеллиуса (закон преломления света): отношение синуса угла падения к синусу угла преломления есть величина постоянная.n – показатель преломления одного вещества относительно другого
v – скорость света в данной среде, [м/с]
Показатель преломления средыsinα/sinγ = nОтношение синуса угла падения к синусу угла преломления есть величина постоянная.n – показатель преломления среды
Формула оптической силы линзыD = 1/FОптическая сила линзы – способность линзы преломлять лучи.D – оптическая сила линзы, [дптр]
F – фокусное расстояние линзы, [м]
Формула тонкой линзы1/F = 1/d + 1/fF – фокусное расстояние линзы, [м]
d – расстояние от предмета до линзы, [м]
f – расстояние от линзы до изображения, [м]
СТРОЕНИЕ АТОМА И АТОМНОГО ЯДРА
Массовое числоM = Z + NM – массовое число
Z – число протонов (электронов), зарядовое число
N – число нейтронов
Формула массы ядраМя = МА – ZmeMя – масса ядра, [кг]
МА – масса изотопа , [кг]
me – масса электрона, [кг]
Формула дефекта масс∆m = Zmp+ Nmn – MЯДефект масс – разность между суммой масс покоя нуклонов, составляющих ядро данного нуклида, и массой покоя атомного ядра этого нуклида.∆m – дефект масс, [кг]
mp – масса протона, [кг]
mn – масса нейтрона, [кг]
Формула энергии связиЕсвязи = ∆m c2Энергия связи ядра – минимальная энергия, необходимая для того, чтобы разделить ядро на составляющие его нуклоны (протоны и нейтроны).Есвязи – энергия связи, [Дж]
m – масса, [кг]
с = 3·108м/с – скорость света
Альфа распадM/Z * X → 4/2 * α + M/Z — 4/2 * Y

Официальный сайт — Ученикам- учитесь!


     1. Физика 
 

 1.1 Формулы по физике
    Формулы по кинематике
          Формулы по динамике, законам сохранения в механике
          Формулы по молекулярной физике
          Формулы по электростатике и постоянному току
          Формулы по магнитному полю, механическим колебаниям
           Формулы по электромагнитным колебаниям, оптике,ТО
        Формулы по квантовой и атомной физике. жидкостям

  1.2 Приблизительные вычисления и погрешности
         О приближенных вычислениях
         О погрешностях
         Погрешности и построения графиков

 

    
   1.3 Задания для дополнительной работы
                 7-9 классы
 
Механическое движение. Плотность
 Механическая работа и мощность      
 
Давление твердых тел, жидкостей и газов
 Архимедова сила       Кинематика             Динамика                       
Законы сохранения механической энергии и импульса
 Механические колебания и волны
 Внутренняя энергия        Изменение агрегатных состояний вещества
 Электрический ток             Работа и мощность тока
Оптические явления              Гравитационные явления
Ответы 7кл        Ответы 8 кл    Ответы 9 кл-1        Ответы 9 кл-2
Справочные данные
        

 

2. Английский язык

 Сайт для родителей второклашек, изучающих английский язык
    1.2 Формулы по физике 
    Формулы по кинематике
          Формулы по динамике, законам сохранения в механике
          Формулы по молекулярной физике
          Формулы по электростатике и постоянному току
          Формулы по магнитному полю, механическим колебаниям
           Формулы по электромагнитным колебаниям, оптике,ТО 
        Формулы по квантовой и атомной физике. жидкостям 

  1.3 Приблизительные вычисления и погрешности
         О приближенных вычислениях
         О погрешностях
         Погрешности и построения графиков

   Физика 
 1.1 Сдаем ЕГЭ
        
 Демонстрационный вариант -2013 
    Тренировочные варианты ЕГЭ-2013
    
Тренировочные варианты ЕГЭ-2012
 1.2 Формулы по физике 
    Формулы по кинематике
          Формулы по динамике, законам сохранения в механике
          Формулы по молекулярной физике
          Формулы по электростатике и постоянному току
          Формулы по магнитному полю, механическим колебаниям
           Формулы по электромагнитным колебаниям, оптике,ТО 
        Формулы по квантовой и атомной физике. жидкостям 

  1.3 Приблизительные вычисления и погрешности
         О приближенных вычислениях
         О погрешностях
         Погрешности и построения графиков

 

Основные формулы по физике по 9 класс. Все, что нужно знать! :: SYL.ru

Физика — строгая техническая наука. Порой не у всех получается успевать в этой дисциплине в школьные годы. Тем более, что не каждый школьник обладает логическим и техническим складом ума, а физику в школе принуждают учить абсолютно каждого. Формулы из учебника могут не укладываться в голове. В данной статье мы рассмотрим основные формулы по физике по 9 класс по механике.

Механика

Начать стоит с самых основных и простейших законов в физике. Как известно, такая обширная тема, как механика состоит из трех параграфов:

  1. Статика.
  2. Динамика.
  3. Кинематика.

Кинематика изучается в 10 классе, поэтому рассматривать ее в рамках данной статьи мы не будем.

Статика

Ее следует изучать последовательно, начиная с простых формул статики. А именно с формул давления, момента инерции тел вращения и момента силы. Формулы по физике 9 класса с пояснениями будут наглядно представлены ниже.

Давление — мера силы, действующая на площадь поверхности тела, измеряется в Паскалях. Давление рассчитывается отношением силы к площади, поэтому формула будет выглядеть максимально просто:

Момент инерции тел вращения — это мера инертности во вращательном движении тела вокруг себя самого, или, строго говоря, произведение массы тела на его радиус, возведенный в квадрат. Соответствующая формула:

Моментом силы (или как многие называют — вращательным моментом) называют силу, приложенную к твердому телу и создающую вращение. Это векторная величина, которая также может иметь отрицательный знак, измеряется в метрах умноженных на Ньютон. В каноничном представлении формула подразумевает собой произведение силы, приложенной к телу и расстояния (плечо силы), формула:

Динамика

Формулы по физике 7-9 класса с пояснениями по динамике — наш следующий этап. Собственно, это самый большой и самый значимый раздел механики. Все тела подвержены движению, даже находясь в состоянии покоя на них действуют некоторые силы, провоцируя на движение. Важные понятия, которые следует изучить перед вниканием в динамику — путь, скорость, ускорение и масса.

Первым делом, конечно же, стоит изучить законы Ньютона.

Первый закон Ньютона — это определение, не имеющее формулы. Он гласит, что тело либо находится в состоянии покоя, либо же движется, но только лишь после того, как все силы, сконцентрированные на нем, будут сбалансированы.

Второй и самый известный закон Ньютона гласит об ускорении тела в зависимости от приложенной к нему силе. В формуле также фигурирует масса объекта, к которому приложена сила.

Обратите внимание, что формула выше записана в скалярном виде — сила и ускорение в векторном могут иметь отрицательный знак, это нужно учитывать.

Третий закон Ньютона: сила действия равна силе противодействия. Все, что нужно знать из этого закона, это то, что каждая сила имеет в противовес такую же силу, только направленную в обратную сторону, таким образом соблюдается баланс на нашей планете.

Теперь же рассмотрим другие силы, действующие в рамках динамики, а это сила тяжести, упругости, трения и сила трения качения. Все они являются векторными и могут быть направлены в любые стороны, также в совокупности способны образовывать системы: складываться и вычитаться, умножаться или делиться. Если силы, направленные не параллельно друг другу, то в вычисления нужно будет использовать косинус угла между ними.

Формулы по физике 9 класса включают в свою программу также закон всемирного тяготения и космические скорости, которые каждый школьник должен знать.

Закон всемирного тяготения — это закон уже небезызвестного нам Исаака Ньютона, фигурирующий в его классической теории. По сути, он оказался революционным: закон утверждает, что любое тело, находящееся в гравитационном поле Земли, притягивается к его ядру. И это действительно так.

Космические скорости

Первая космическая скорость необходима для выхода на орбиту Земли (численно равна 7,9 км/с), а вторая космическая скорость нужна для преодоления гравитационного притяжения, чтобы выйти не только за орбиту, но и позволить объекту двигаться не по круговой траектории. Она равна 11,2 км/с соответственно. Важно, что обе космические скорости были преодолены человечеством, и благодаря им сегодня возможны полеты в космос. Формулы по физике по 9 класс не предполагают третью и четвертую космические скорости, однако они также существуют.

Вывод

В этой статье были рассмотрены основные формулы по физике по 9 класс. Их изучение открывает возможности школьнику познавать более сложные разделы физики, такие как электричество, магнетизм, звук или молекулярную теорию. Не зная механику, невозможно понять остальную физику, механика является основополагающей частью этой науки на сегодняшний день. Формулы по физике по 9 класс также необходимы для прохождения государственного экзамена ОГЭ по физике, их краткое содержание и написание обязан знать каждый выпускник 9-го класса, поступающий в технический колледж. Запомнить их не составляет труда.

Конспект урока физики в 9 классе «Решение задач по теме «Кинематика»

Дата: Урок физики в 9 классе

Тема урока: Решение задач по теме “Кинематика”.

Учитель: Казьмина Валентина Федосовна

Цели урока:

Образовательная

  • научить применять имеющиеся знания к различным теоретическим и практическим заданиям;

  • повторить основные определения, понятия и формулы в ходе урока;

  • совершенствовать навыки решения качественных и расчетных задач;

  • закрепить межпредметные связи с алгеброй (чтение графиков)

Воспитательная

  • формирование навыков коллективной работы;

  • формирование сознательной дисциплины;

  • воспитание уважительного отношения друг к другу, к точке зрения товарищей;

  • повышение познавательной деятельности и активности учащихся;

  • воспитание умения достойно проигрывать, анализировать пробелы и промахи

  • выработка системы подготовки к ГИА.

Развивающая

  • развитие интереса к физике;

  • развитие речи учащихся,

  • развитие коммуникативных способностей;

  • развитие культуры общения;

В ходе урока планируется формирование у школьников ключевых компетенций:

1. в познавательной деятельности:

  • использование для познания окружающего мира естественнонаучных методов наблюдения, измерения, эксперимента;

  • формирование умений применять законы кинематики

  • овладение адекватными способами решения теоретических и экспериментальных задач;

2. в информационно-коммуникативной деятельности:

  • способности понимать точку зрения собеседника и признавать право на иное мнение.

3. в рефлексивной деятельности:

  • владение навыками оценки своей деятельности, умением предвидеть возможные результаты своих действий.

4. здоровьесбережения:

  • знание и соблюдение правил техники безопасности; четкая смена видов деятельности, опора на эмоциональный подъём и интерес при решении задач.

Оборудование и материалы : листы контроля, мультимедийная доска, ПК, тестовые задания, раздаточный материал, карточки –задания. Презентации 1 и 2

Этапы урока:

  • постановка цели урока и мотивация учебной деятельности;

  • воспроизведение и коррекция опорных знаний;

  • выполнение заданий на карточках;

  • физкульпауза

  • электронное тестирование

  • чтение графика;

  • подведение итогов урока (оценки, домашнее задание, рефлексия).

Тип урока: урок обобщения пройденного материала и применения знаний к решению задач.

ХОД УРОКА.

1. Организационный момент —3 минуты.

Сегодня у нас урок будет очень насыщенным.Вы повторите пройденный материал, решите задачи, пройдете электронное тестирование, совершенствуете свое умение читать графики.

Мы с вами закончили изучать тему «Кинематика». Повторим основные понятия кинематики и тем самым осуществить еще один шаг в подготовке к ГИА.

Чтобы наша работа прошла четко и слаженно я приготовила листы контроля, где отражены основные этапы урока .

  1. Этап повторения пройденного материала (7 минут).

А начнем мы свой урок с разминки ( Урок сопровождается показом ПОЛОЖЕНИЕ ТЕЛА В ПРОСТРАНСТВЕ.ppsx)

2-й слайд:

3 слайд: определите координаты зайца и автомобиля

На 4-м слайде определите Координаты лодки:

5-й слайд:

6-й слайд:

Итак, мы повторили основные понятия. Теперь приступим к следующему этапу.

  1. Выполнение заданий на карточках (10 минут).

1-е задание:Раздаю учащимся следующие задания:

  1. Вы совершаете прогулку: 3 километра к востоку, 2 километра к северу, 3 километра к западу. На каком расстоянии от исходной точки вы окажетесь. (На 2 километра к северу).

  2. В субботу до возвращения в гараж автобус сделал 10 рейсов, а в воскресенье – 15 рейсов. В какой из этих дней автобус проехал больший путь? Совершил большее перемещение? (Больший путь – в воскресенье, а перемещение и в субботу и в воскресенье равно 0, т.к. начальная и конечная точки траектории совпадали).

  3. Какую форму должна иметь траектория точки, чтобы пройденный ею путь мог равняться перемещению? (Траектория – прямая).

  4. Лётчик – спортсмен сумел посадить самолёт на крышу легкового автомобиля. При каком условии это возможно? (Когда скорость самолёта относительно автомобиля равна 0; скорости самолёта и автомобиля относительно Земли равны).

  5. Какая стрелка движется быстрее: секундная на ручных часах или минутная на башенных? (Секундная движется быстрее).

  6. Из точки А в восточном направлении до точки В самолет долетел за 80 минут, а из точки В в западном направлении до точки А — за 1 час 20 минут. Почему? Пояснение: Отвечая на этот вопрос, часто пытаются объяснить разницу в скорости полета различными причинами (направлением ветра, скоростью вращения Земли и т. п.), не обращая внимания на то, что 80 минут и 1 час 20 минут это одно и то же.

  7. Утром из Раздольного в Симферополь выходит автобус с пассажирами. Часом позже из Симферополя в Раздольное выезжает велосипедист и едет по тому же шоссе, но, конечно, значительно медленнее, чем автобус. Когда пассажиры автобуса и велосипедист встретятся, то кто из них будет дальше от Симферополя? Пояснение: Встретившиеся путешественники находятся в одном месте, и, следовательно, на одинаковом расстоянии от Москвы.

  8. Если в 12 часов ночи идет дождь, то можно ли ожидать, что через 72 часа будет солнечная погода? Пояснение: Через 72 часа, т. е. ровно через трое суток, будет ночь, значит, солнечной погоды быть не может.

  1. Физкультминутка (учащиеся разминаются, выполняют простые упражнения для гимнастики глаз.) – 3 минуты.

2 задание : Работа с формулами: два этапа

Первый этап: На доске написаны формулы Но в них пропущены некоторые физические величины. Вы должны вспомнить и написать недостающие величины, чтобы получились верные формулы, которые вы изучили. (3 минуты)

Второй этап: Предлагаю Вам выполнить тест, где вы покажете свои знания формул, определений, единиц измерения физических величин. (вопросы к тесту в приложении 1) (12 минут)

Тест

Задание #1

Вопрос:

Изменение положения тела относительно другого тела с течением времени называют:

1. пройденным путем

2. траекторией

3. механическим движением.

Выберите один из 3 вариантов ответа:

1) 1

2) 2

3) 3

Задание #2

Вопрос:

Линию, которую описывает тело при своем движении, называют:

1. пройденным путем

2. траекторией

3. механическим движением.

Выберите один из 3 вариантов ответа:

1) 1

2) 2

3) 3

Задание #3

Вопрос:

Направленный отрезок прямой ,соединяющий начальное положение тела с последующим, называется :

1. пройденным путем

2. траекторией

3. перемещение.

Выберите один из 3 вариантов ответа:

1) 1

2) 2

3) 3

Задание #4

Вопрос: Какую физическую величину можно определить с помощью формулы

1. перемещение

2. ускорение

3. силу

Выберите один из 3 вариантов ответа:

1) 1

2) 2

3) 3

Задание #5

Вопрос:

Величина, которая измеряется в метрах в секунду (м/с)

1. ускорение

2. путь

3. скорость

Выберите один из 3 вариантов ответа:

1) 1

2) 2

3) 3

Задание #6

Вопрос:

По какой формуле рассчитывается путь при равноускоренном движении?

1.

2. v t

3.

Выберите один из 3 вариантов ответа:

1) 1

2) 2

3) 3

Задание #7

Вопрос:

. Формула второго закона Ньютона

1. F = а m

2.

3. F1 = — F2

Выберите один из 3 вариантов ответа:

1) 1

2) 2

3) 3

Задание #8

Вопрос:

Формула скорости при равноускоренном движении

1. v0 + a t

2.

3.

Выберите один из 3 вариантов ответа:

1) 1

2) 2

3) 3

Задание #9

Вопрос:

Какое уравнение соответствует зависимости координаты от времени при равноускоренном движении ?

1.

2. v = v0 + a t

3.

Выберите один из 3 вариантов ответа:

1) 1

2) 2

3) 3

Задание #10

Вопрос:

Автомобиль движется равномерно по мосту со скоростью 18 км/ч. За какое время он пройдет мост, если длина моста 480 м?

Выберите один из 3 вариантов ответа:

1) 96 c

2) 96 ч

3) 27 с

Ответы:

1) (1 б.) Верные ответы: 3;

2) (1 б.) Верные ответы: 2;

3) (1 б.) Верные ответы: 3;

4) (1 б.) Верные ответы: 2;

5) (1 б.) Верные ответы: 3;

6) (1 б.) Верные ответы: 1;

7) (1 б.) Верные ответы: 1;

8) (1 б.) Верные ответы: 1;

9) (1 б.) Верные ответы: 3

10) (1 б.) Верные ответы: 1;

5. задание . Проверка умения читать графики скорости равномерного и равноускоренного движения.(5 минут)

В этом виде заданий нужно показать свои умения находить различные величины по графику скорости. Начинаю показывать Равноускоренное движение1.ppsx

Дан график скорости ( см.слайд 8). Попробуйте ответить на следующие вопросы :

  1. Начальная скорость тела. (2)

  2. Скорость тела через 2 секунды после начала движения. (5)

  3. Сколько времени двигалось тело равномерно? (4)

  4. Ускорение тела на первом участке (1/2)

  5. Ускорение тела на втором участке (0)

  6. Ускорение тела на третьем участке (1/4)

  7. Путь, пройденный телом на первом участке (12)

  8. Путь, пройденный телом на втором участке (16)

6. ИТОГИ УРОКА. ( 2 минуты)

Изучая кинематику, вы должны были овладеть ее основными понятиями и законами. Эти законы мы сегодня повторили и наблюдали их применение к решению задач. Но особенно важно знать применение законов кинематики в технике. Это поможет вам в дальнейшем понять принципы устройства и работы тех машин и механизмов, с которыми придется иметь дело на производстве и в быту.

Итак, сегодня на уроке мы получили следующие результаты (подводятся итоги выставляются оценки за активную работу на уроке). Класс сегодня работал активно. Спасибо. Молодцы!

7. РЕФЛЕКСИЯ

Понравился ли сегодняшний урок?

Я в сомнениях….. Да!

Домашнее задание:

Notes — Class 9 Physics — ClassNotes

  • 1 кв. Определите кинематику?
  • 2 кв. В чем разница между отдыхом и движением?
  • 3 кв. Определить окружение?
  • 4 кв. Перечислите типы движения?
  • 5 квартал. Описать движение переводчика на примерах?
  • 6 кв. Опишите различные типы движения переводчика?
  • Q7. Описать вращательное движение на примерах?
  • 8 кв. Вы можете указать на некоторые различия в круговом и вращательном движении?
  • Когда говорят, что тело находится в состоянии покоя?
  • Приведите пример тела, которое одновременно находится в состоянии покоя и находится в движении.
  • Укажите тип движения в каждом из следующих пунктов:
  • Q9. Опишите колебательные движения с помощью примеров?
  • Q10. Отличить скаляры от векторов?
  • Q11. Как можно графически представить векторные величины?
  • Q12. Дайте определение термину «позиция»?
  • Q13. Объясните разницу между расстоянием и смещением?
  • Q14. В чем разница между скоростью и скоростью?
  • Q15. Определите равномерную скорость.
  • Q16. Определить переменную скорость?
  • Q17. Определите среднюю скорость?
  • Q18. Определить равномерную скорость?
  • Q19. Определить переменную скорость?
  • Q20. Определить среднюю скорость?
  • Q21. Определите ускорение?
  • Q23. Как отличить положительное ускорение от отрицательного?
  • Q24. Что вы имеете в виду под графиком, переменными, независимой величиной и зависимой величиной?
  • Q25. Какова цель графика расстояние-время? Как это построено?
  • Q26.Нарисуйте график расстояния-времени для тела в состоянии покоя. Как вы определите скорость тела по этому графику?
  • Q27. Построить и интерпретировать график расстояния-времени для тела, движущегося с постоянной скоростью?
  • Q28. Нарисуйте график расстояние-время для тела, движущегося с переменной скоростью?
  • Q29. Что вы имеете в виду под графиком скорость-время?
  • Q30. Нарисуйте график скорость-время для тела, движущегося с постоянной скоростью?
  • OR
  • Какой была бы форма графика скорость-время тела, движущегося с постоянной скоростью?
  • Q31.Нарисуйте график скорость-время для тела, движущегося с равномерно меняющейся скоростью?
  • OR
  • Какой была бы форма графика скорость-время тела, движущегося с равномерно изменяющейся скоростью?
  • Q32. Нарисуйте график скорости-времени для расстояния, на которое движется движущийся объект?
  • OR
  • Какой была бы форма графика скорость-время для расстояния, пройденного движущимся объектом?
  • Q33. Опишите назначение различных уравнений движения?
  • Q34. Выведите первое уравнение движения для равноускоренного прямолинейного движения.
  • OR
  • Какое уравнение движения устанавливает взаимосвязь между vf, vi, a и t, взаимосвязь между этими величинами.
  • OR
  • Докажите, что vf = vi + at.
  • OR
  • Выведите уравнение движения, которое не зависит от расстояния S.
  • Q35. Выведите второе уравнение движения для равноускоренного прямолинейного движения.
  • OR
  • Какое уравнение движения устанавливает взаимосвязь между S, a, Vi и Vf?
  • OR
  • Выведите уравнение движения, не зависящее от t.
  • OR
  • Вывести второе уравнение движения?
  • OR
  • Докажите, что S = v1 t + ½ at2
  • Q36. Выведите третье уравнение движения для равноускоренного прямолинейного движения.
  • OR
  • Какое уравнение движения устанавливает взаимосвязь между S, a, Vi и Vf?
  • OR
  • Выведите уравнение движения, не зависящее от t.
  • OR
  • Вывести третье уравнение движения?
  • OR
  • Докажите, что 2aS = vf2 — vi2
  • Q37.Бросьте предмет с некоторой высоты и наблюдайте за его движением. Его скорость увеличивается, уменьшается или остается постоянной по мере приближения к земле?
  • Q38. Объясните движение свободно падающих тел?
  • Q39. Написать уравнения движения тел, движущихся под действием силы тяжести?

Банкноты

1 кв. Определите кинематику?

Ответ: Кинематика:

Кинематика — это исследование движения объекта без обсуждения причины движения.

2 кв. В чем разница между отдыхом и движением?

Ответ: Разница между покоем и движением:

Остальное:

Тело считается покоящимся, если оно не меняет своего положения по отношению к окружающей среде.

Движение:

Тело считается движущимся, если оно меняет свое положение по отношению к окружающей среде.

Состояние покоя или движения тела относительное. Например, пассажир, сидящий в движущемся автобусе, находится в состоянии покоя, потому что он / она не меняет своего положения по отношению к другим пассажирам или объектам в автобусе. Но для наблюдателя снаружи автобуса пассажиры и предметы внутри автобуса находятся в движении.

3 кв. Определить окружение?

Ответ: Окрестности:

Окрестности — это места по соседству с различными предметами.Аналогично

4 кв. Перечислите типы движения?

Ответ: Типы движения:

Есть три типа движения.

  1. Поступательное движение (линейное, случайное и круговое)
  2. Вращательное движение
  3. Вибрационное движение (движение вперед и назад)

Q5. Описать движение переводчика на примерах?

Ответ: Переводное ходатайство:

При поступательном движении тело движется по прямой без вращения.Линия может быть прямой или изогнутой.

Примеры:

Всадники, движущиеся в колесе обозрения, также совершают поступательное движение. Их движение происходит по кругу без вращения.

Q6. Опишите различные типы движения переводчика?

Ответ: Типы движений переводчика:

Движения переводчика можно разделить на линейные, круговые и произвольные.

Прямолинейное движение тела известно как его линейное движение.

Примеры:

Движение объектов, таких как автомобиль, движущееся по прямой и ровной дороге, является линейным движением.

Самолеты, летящие прямо в воздухе, и объекты, падающие вертикально вниз, также являются примером линейного движения.

Движение объекта по круговой траектории известно как круговое движение

Примеры:

Камень, привязанный к концу веревки, можно заставить вращаться. Камень движется по кругу и, следовательно, имеет круговое движение.

Игрушечный поезд, движущийся по круговой колее. Велосипед или автомобиль, движущиеся по круговой дорожке, обладают круговым движением. Движение Земли вокруг Солнца и движение Луны вокруг Земли также являются примерами круговых движений.

Неупорядоченное или нерегулярное движение объекта называется случайным движением.

Примеры:

Движение насекомых и птиц нерегулярное. Таким образом, движение насекомых и птиц — это случайное движение.

Движение частиц пыли или дыма в воздухе также является случайным движением.

Броуновское движение молекул газа или жидкости по зигзагообразной траектории также является примером случайного движения.

Q7. Описать вращательное движение на примерах?

Ответ: Вращательное движение:

Вращательное движение тела вокруг своей оси называется его вращательным движением.

Примеры:

Вершина вращается вокруг своей оси, проходящей через нее, и, таким образом, обладает вращательным движением.Ось — это линия, вокруг которой вращается тело. При круговом движении точка, вокруг которой движется тело, находится вне тела. При вращательном движении линия, по которой движется тело, проходит через само тело.

Движение колеса вокруг своей оси и движение рулевого колеса являются примерами вращательного движения. Движение Земли вокруг Солнца является круговым, а не вращательным. Однако движение Земли вокруг своей географической оси, вызывающее день и ночь, является вращательным.

Q8. Вы можете указать на некоторые различия в круговом и вращательном движении?

Ответ: Различия в круговом и вращательном движении:

Любое вращение, как будто на оси, является вращательным движением. Любое вращательное движение с фиксированными радиусом инерционной длины и осью вращения является круговым движением. И в этом разница. Круговое движение — это просто частный случай вращательного движения. То есть нет ограничений по фиксированной оси и радиусу для вращательного движения, но есть для кругового движения.

Например, , все планеты вращаются вокруг своих солнц. Но большинство орбит имеют эллиптическую форму, поэтому ось вращения (их две в эллипсе) и радиусы вращения меняются по мере их движения. Итак, большинство, если не все, планеты не имеют кругового движения.

Примечание:

Длина вращения:

Длина, которая представляет собой расстояние во вращающейся системе между точкой, вокруг которой она вращается, и точкой, в которую или от которой передача энергии имеет максимальный эффект.

Мини-упражнения

  1. Когда говорят, что тело находится в состоянии покоя?

Ответ: Тело считается покоящимся, если оно не меняет своего положения относительно окружающей среды.

  1. Приведите пример тела, которое одновременно находится в состоянии покоя и находится в движении.

Ответ: Движение и покой — понятия относительные. Абсолютного покоя нет. Мы можем определить состояние покоя или движения только в отношении другого объекта или точки в пространстве, взятых за точку отсчета.

Примеры:

  • Человек в поезде считает, что находится в состоянии покоя относительно попутчиков или стен поезда. Но когда он смотрит на улицу, он обнаруживает, что движется к деревьям снаружи.
  • Пассажир, сидящий в движущемся автобусе, находится в состоянии покоя, потому что он / она не меняет своего положения по отношению к другим пассажирам или объектам внутри автобуса.Но для наблюдателя снаружи автобуса пассажиры и предметы внутри автобуса находятся в движении.

  1. Укажите тип движения в каждом из следующих пунктов:

  • Шар, движущийся вертикально вверх.

Ответ: Линейное движение.

  • Ребенок, спускающийся с горки.

Ответ: Линейное движение.

  • Движение игрока на футбольном поле.

Ответ: Случайное движение.

  • Полет бабочки.

Ответ: Случайное движение .

  • Спортсмен, бегущий по круговой дорожке.

Ответ: Круговое движение .

Ответ: Круговое движение.

Ответ: Вибрационное движение.

Q9. Опишите колебательные движения с помощью примеров?

Ответ: Вибрационное движение:

Движение тела взад и вперед относительно его среднего положения известно как колебательное движение.

Примеры:

Считайте движение, если ребенок качается. Когда его толкают, качели перемещаются взад и вперед относительно своего среднего положения.Движение ребенка повторяется от одной крайности до другой с качелями. Такой тип движения называется колебательным.

Возвратно-поступательное движение маятника часов относительно его среднего положения, это называется колебательным движением.

Младенец в колыбели, двигающийся туда-сюда, туда-сюда, движение молотка электрического колокольчика и движение струны ситара — вот некоторые из примеров колебательного движения.

Q10.Отличить скаляры от векторов?

Ответ: различать скаляры и векторы

Скаляры Векторы
Скалярная величина полностью описывается только своей величиной. Векторная величина полностью описывается величиной и направлением
Примеры: Примеры скаляров: масса, длина, время, скорость, объем, работа, энергия, плотность, мощность, электрический заряд, давление, площадь, температура. Примеры: Примеры векторов: скорость, смещение, сила, импульс, крутящий момент, вес, электрический потенциал и т. Д.

Q11. Как можно графически представить векторные величины?

Ответ: Представление векторов (символическое представление вектора):

Чтобы отличить вектор от скалярной величины, мы обычно используем жирные буквы для обозначения векторных величин, например F , a, d или черту или стрелку над их символами, например

.

Векторное представление / Графическое представление вектора:

Проведена прямая линия со стрелкой на одном конце.Длина линии согласно некоторому подходящему масштабу представляет величину, а острие стрелки указывает направление вектора.

Q12. Дайте определение термину «позиция»?

Ответ: Должность:

Термин «положение» описывает положение места или точки относительно некоторой контрольной точки, называемой исходной точкой.

Например:

Вы хотите описать расположение вашей школы, не выходя из дома.Пусть школа будет обозначена буквой S, а дом — буквой H. Положение вашей школы от вашего дома будет обозначено прямой HS в направлении от H к S.

Q13. Объясните разницу между расстоянием и смещением?

Отв. Разница между расстоянием и перемещением:

Расстояние Рабочий объем
Длина пути между двумя точками называется расстоянием между этими точками. Смещение — это кратчайшее расстояние между двумя точками, имеющее величину и направление.
Расстояние — это скалярная величина. Смещение — это векторная величина.
Расстояние обозначается буквой «S»

S = VT

Его единица в системе СИ — метр (м).

Рабочий объем обозначается « d ».

D = VT

Его единица в системе СИ — метр (м).

Расстояние S (пунктирная линия) и смещение d (темная линия) от точек A до B,

Q14.В чем разница между скоростью и скоростью?

Отв. Разница между скоростью и скоростью:

Скорость Скорость
Расстояние, пройденное объектом за единицу времени, называется его скоростью.

Расстояние = скорость x время

или S = ​​vt

Скорость перемещения тела называется его скоростью.

V = d / t или d = vt

Скорость — это скалярная величина. Скорость — это векторная величина.
Sl. Единица измерения скорости — метр в секунду. (мс -1 ) Sl. Единица измерения скорости такая же, как у скорости, то есть метр в секунду. (мс -1 )

ЗНАЕТЕ ЛИ ВЫ

Какое самое быстрое животное на земле?

Falcon может летать со скоростью 200 км / ч -1

Гепард может бегать со скоростью 70 км / ч -1

ЛИДАР-пушка — это скорость обнаружения света и дальности.Он использует время, затрачиваемое лазерным импульсом, на серию измерений расстояния транспортного средства до пушки. Затем данные используются для расчета скорости автомобиля.

Парашютист достигает равномерной скорости, называемой предельной скоростью, с которой он приближается к земле.

Q15. Определите равномерную скорость.

Отв. Равномерная скорость:

Тело имеет равномерную скорость, если оно преодолевает равные расстояния за равные промежутки времени, какими бы короткими ни были эти промежутки.

Q16. Определить переменную скорость?

Отв. переменная скорость:

Если тело преодолевает неравные расстояния за равный интервал времени, однако этот интервал может быть небольшим, скорость тела считается переменной.

Q17. Определите среднюю скорость?

Отв. Средняя скорость:

Соотношение между пройденным расстоянием и общим временем называется средней скоростью.

Q18. Определить равномерную скорость?

Отв. Равномерная скорость:

Тело имеет равномерную скорость, если оно преодолевает равное перемещение за равные промежутки времени, какими бы короткими ни были эти промежутки.

Q19. Определить переменную скорость?

Отв. переменная скорость:

Если скорость или направление меняются со временем, то скорость такого тела называется переменной.

Q20. Определить среднюю скорость?

Отв. Средняя скорость:

Отношение между смещением и временем известно как средняя скорость

Q21. Определите ускорение?

Отв. Разгон:

Ускорение определяется как скорость изменения скорости тела.

Единица ускорения:

Единица измерения ускорения в системе СИ — метр на секунду в квадрате (мс -2 )

ПОЛЕЗНАЯ ИНФОРМАЦИЯ

Ускорение движущегося объекта происходит в сторону увеличения скорости его скорости. Ускорение объекта противоположно направлению скорости, если его скорость уменьшается.

Q22.Определить равномерное ускорение?

Отв. Равномерное ускорение:

Тело имеет равномерное ускорение, если оно имеет равные изменения скорости в равные промежутки времени, какими бы короткими ни были эти промежутки.

Q23. Как отличить положительное ускорение от отрицательного?

Ответ: Положительное ускорение:

Ускорение тела положительное, если его скорость увеличивается со временем.Направление этого ускорения такое же, в каком движется тело, не меняя своего направления.

Отрицательное ускорение / замедление или замедление:

Ускорение тела отрицательное, если скорость тела уменьшается. Направление отрицательного ускорения противоположно направлению движения тела. Отрицательное ускорение также называется замедлением или замедлением.

ЗНАЕТЕ ЛИ ВЫ

График также можно использовать в повседневной жизни, например, чтобы показать рост / снижение экспорта по годам, количество осадков по месяцам, данные о температуре пациента или количество пробежек, набранных командой, и т. Д.

Q24. Что вы имеете в виду под графиком, переменными, независимой величиной и зависимой величиной?

Ответ: График:

График — это наглядный способ представления информации о взаимосвязи между различными величинами.

Переменные:

Величины, между которыми построен график, называются переменными.

Независимое количество:

Одна из величин называется независимой величиной.

Зависимое количество:

Значения, значения которых зависят от независимой величины, называются зависимой величиной.

Q25. Какова цель графика расстояние-время? Как это построено?

Ответ: График расстояние-время:

Полезно представлять движение объектов с помощью графиков. Термины расстояние и смещение используются как синонимы, когда движение происходит по прямой.Точно так же, если движение по прямой, то скорость и скорость также используются взаимозаменяемо

Примечание:

На графике расстояние-время время отсчитывается по горизонтальной оси, а по вертикальной оси показано расстояние, пройденное объектом.

Q26. Нарисуйте график расстояния-времени для тела в состоянии покоя. Как вы определите скорость тела по этому графику?

Ответ: Объект в состоянии покоя:

На графике, показанном на рисунке, расстояние, на которое объект перемещается со временем, равно нулю.То есть объект покоится. Таким образом, горизонтальная линия, параллельная оси времени на графике расстояние-время, показывает, что скорость объекта равна нулю.

Q27. Построить и интерпретировать график расстояния-времени для тела, движущегося с постоянной скоростью?

Ответ: объект движется с постоянной скоростью:

Скорость объекта называется постоянной, если он преодолевает равные расстояния за равные промежутки времени.График расстояние-время, показанный на рисунке, представляет собой прямую линию.

Его наклон дает скорость объекта.

Рассмотрим две точки A и B на графике.

Скорость объектов = наклон линии AB

Скорость, определенная по графику, составляет 2 мс -1

Q28. Нарисуйте график расстояние-время для тела, движущегося с переменной скоростью?

Ответ: объект движется с переменной скоростью:

Когда объект не преодолевает равные расстояния за равные промежутки времени, его скорость не постоянна.В этом случае график расстояние-время не будет прямой линией, как показано на рисунке. Наклон кривой в любой точке можно определить по наклону касательной в этой точке. Например.

Таким образом, скорость объекта в точке P составляет 3 мс -1

Примечание:

Скорость выше в моменты, когда наклон больше, скорость равна нулю в моменты, когда наклон горизонтальный.

Q29.Что вы имеете в виду под графиком скорость-время?

Ответ: график скорости-времени:

На графике скорость-время время отсчитывается по оси x, а скорость — по оси y.

Q30. Нарисуйте график скорость-время для тела, движущегося с постоянной скоростью?

ИЛИ

Какой была бы форма графика скорость-время тела, движущегося с постоянной скоростью?

Ответ: объект движется с постоянной скоростью

Когда скорость объекта постоянна (4 мс -1 ) во времени, тогда график скорость-время будет горизонтальной линией, параллельной оси времени вдоль оси x.

Прямая линия, параллельная оси времени, представляет постоянную скорость объекта.

Q31. Нарисуйте график скорость-время для тела, движущегося с равномерно меняющейся скоростью?

ИЛИ

Какой была бы форма графика скорость-время тела, движущегося с равномерно меняющейся скоростью?

Ответ: объект движется с равномерно меняющейся скоростью (равномерное ускорение):

Пусть скорость объекта изменяется равномерно.В этом случае скорость изменяется с постоянной скоростью. Таким образом, его график скорости-времени был бы такой прямой линией.

Прямая линия означает, что объект движется с равномерным ускорением. Наклон линии дает величину ее ускорения.

График скорость-время дает отрицательный наклон. Таким образом, объект имеет замедление 0,4 мс -2 .

Q32. Нарисуйте график скорости-времени для расстояния, на которое движется движущийся объект?

ИЛИ

Какой была бы форма графика скорость-время для расстояния, проходимого движущимся объектом?

Ответ: Расстояние, пройденное движущимся объектом:

Область под графиком скорость-время представляет расстояние, пройденное объектом.Если движение равномерное, площадь можно рассчитать, используя соответствующую формулу для геометрических фигур, представленных на графике.

Q33. Опишите назначение различных уравнений движения?

Ответ: Уравнения движения:

Существуют три основных уравнения движения тел, движущихся с равномерным ускорением. Эти уравнения связывают начальную скорость, конечную скорость, ускорение, время и расстояние, пройденное движущимся телом.

Q34. Выведите первое уравнение движения для равноускоренного прямолинейного движения.

ИЛИ

Какое уравнение движения устанавливает взаимосвязь между v f, v i , a и t, взаимосвязь между этими величинами.

ИЛИ

Докажите, что v f = v i + at.

ИЛИ

Выведите уравнение движения, не зависящее от расстояния S.

Ответ: Предположим, что тело движется с начальной скоростью v i , и по прошествии времени t его скорость станет v f . Тогда ускорение a равно

или V f — V i = at

V f = V i + at

Второй метод (графический метод):

Первое уравнение движения:

График скорость-время движения тела показан на рисунке.Наклон линии AB дает ускорение тела.

как

BD = V f , CD = V i и OD = t

Следовательно

или V f — V i = at

V f = V i + at

Q35. Выведите второе уравнение движения для равноускоренного прямолинейного движения.

ИЛИ

Какое уравнение движения устанавливает взаимосвязь между S, a, V i и V f ?

ИЛИ

Выведите уравнение движения, не зависящее от t.

ИЛИ

Вывести второе уравнение движения?

ИЛИ

Докажите, что S = v 1 t + ½ при 2

Ответ: Предположим, что тело движется с начальной скоростью v i и через определенное время t его скорость становится v f , тогда общее расстояние S, пройденное за время t, равно

S = v ср x t

) x т ………………….. (1)

Из первого уравнения движения. V f = V i + at

Подставляем значение V f в уравнение (1).

) х т

) х т

)

S = V.t + ½ при 2

Второй метод (графический метод):

Второе уравнение движения:

На графике скорость-время, показанном на рисунке, общее расстояние S, пройденное телом, равно общей площади OABD под графиком.То есть

Общее расстояние S = площадь (прямоугольник OACD + треугольник ABC)

Площадь прямоугольника OACD = OA x OD

= V i x t

Площадь треугольника ABC = ½ (AC x BC)

= ½ т x при

Т.к. Общая площадь OABD = площадь прямоугольника OACD + площадь треугольника ABC

Подставляя значения в приведенное выше уравнение, получаем

S = V i t + ½ t x при

S = V i t + ½ при 2

Q36.Выведите третье уравнение движения для равноускоренного прямолинейного движения.

ИЛИ

Какое уравнение движения устанавливает взаимосвязь между S, a, V i и V f ?

ИЛИ

Выведите уравнение движения, не зависящее от t.

ИЛИ

Вывести третье уравнение движения?

ИЛИ

Докажите, что 2aS = v f 2 — v i 2

Ответ: Предположим, что тело движется с начальной скоростью v i и через определенное время t его скорость становится v f , тогда общее расстояние S, пройденное за время t, равно

S = v ср x t

) x т ………………….. (1)

Из первого уравнения движения найдите значение t.

V f = V i + at или

Подставляем значение V f в уравнение (1).

2aS = (v f + v i ) x (v f — v i ) по формуле (a + b) (a — b) = a 2 — b 2

2aS = v f 2 — v i 2

Второй метод (графический метод)

Третье уравнение движения:

На графике скорость-время, показанном на рисунке, общее расстояние S, пройденное телом, выражается общей площадью OABD под графиком.

Общая площадь OABD =

или 2S = (OA + BD) x OD

Умножаем обе стороны на BC / OD, получаем: (BC / OD = a)

2S x = (OA + BD) X OD X

2S x = (OA + BD) X BC ……………………. (1)

Подставляя значение в вышеприведенное уравнение (1), получаем

2S x a = (V i + V f ) x (V f — V i )

2aS = V f 2 + V i 2

ПОЛЕЗНАЯ ИНФОРМАЦИЯ

1 мс -1 = 0.001 км x 3600 = 3,6 км / ч -1

Таким образом, умножьте скорость в мс -1 на 3,6, чтобы получить скорость в км / ч -1 например,

20 мс -1 = 20 x 3,6 км / ч -1 = 72 км / ч -1

1 км / ч -1 = мс -1

Таким образом, умножьте скорость в км / ч -1 на, чтобы получить скорость в мс -1 , например

50 км / ч -1 = 50 x мс -1 = 13,88 мс -1

Умножьте ускорение в мс -2 на = 12960, чтобы получить его значение в км / ч -2

Разделите ускорение в км / ч -2 на 12960, чтобы получить его значение в мс -2

Q37.Бросьте предмет с некоторой высоты и наблюдайте за его движением. Его скорость увеличивается, уменьшается или остается постоянной по мере приближения к земле?

Ответ: Скорость объекта увеличится из-за земного притяжения. Поэтому для свободно падающих тел g положительна.

Q38. Объясните движение свободно падающих тел?

Ответ: Движение свободно падающих тел:

Ускорение свободно падающих тел называется ускорением свободного падения.

Обозначается g. На поверхности Земли его значение составляет примерно 10 мс -2 .

Для тел, свободно падающих, g положительна, а для тел, движущихся вверх, отрицательна.

Галилей был первым ученым, заметившим, что все свободно падающие объекты имеют одинаковое ускорение независимо от их масс. Он сбрасывал с Пизанской башни различные предметы разной массы. Он обнаружил, что все они достигают земли одновременно.

Q39. Написать уравнения движения тел, движущихся под действием силы тяжести?

Ответ: Уравнения движения тел, движущихся под действием силы тяжести:

  1. V f = V i + gt
  2. h = V i t + ½ gt 2

РЕЗЮМЕ

  1. Покой: Тело считается покоящимся, если оно не меняет своего положения относительно окружающей среды.
  2. Движение: Говорят, что тело находится в движении, если оно меняет свое положение относительно окружающей среды.
  3. Покой и движение всегда относительны. Абсолютного покоя или абсолютного движения не существует.
  4. Движение можно разделить на следующие три типа:

Поступательное движение: тело движется без вращения.

Вращательное движение: тело вращается вокруг своей оси.

Вибрационное движение: при котором тело движется взад и вперед относительно своего среднего положения.

  1. Скаляры: физические величины, которые описываются только своей величиной, называются скалярами.
  2. Векторы: физические величины, которые описываются своей величиной и направлением, называются векторами.
  3. Позиция: Позиция означает местоположение определенного места или объекта от контрольной точки.
  4. Скорость: расстояние, которое мальчик преодолевает в любом направлении за единицу времени, называется скоростью.
  5. Равномерная скорость: если скорость тела не меняется со временем, то его скорость одинакова.
  6. Средняя скорость: Средняя скорость тела — это отношение общего пройденного расстояния к общему затраченному времени
  7. Скорость: мы определяем скорость как скорость изменения смещения или скорости в определенном направлении.
  8. Средняя скорость: Средняя скорость тела определяется как отношение его чистого смещения к общему времени.
  9. Равномерная скорость: Если тело совершает равные перемещения за равные промежутки времени, каким бы малым ни был этот интервал, его скорость называется равномерной.
  10. Ускорение: Скорость изменения скорости тела называется ускорением.
  11. Равномерное ускорение: Тело имеет равномерное ускорение, если оно имеет равные изменения скорости в равные промежутки времени, каким бы малым ни был этот интервал.
  12. График: График — это наглядный способ описания информации о том, как различные величины связаны друг с другом.
  13. Наклон графика расстояние-время показывает скорость тела.
  14. График «расстояние-время»: графики «расстояние-время» предоставляют полезную информацию о движении объекта.Наклон графика смещения-время дает скорость тела.
  15. Расстояние, пройденное телом, равно площади под графиком скорость-время.
  16. График «скорость-время»: график «скорость-время» также полезен для изучения движения по прямой.
  17. График скорости-времени: расстояние, пройденное телом, также можно найти в области под графиком «Скорость-время», если движение идет по прямой.
  18. Уравнения движения для равноускоренного движения:
  • V f = V i + at
  • S = V i t + ½ при 2
  • 2aS = V f 2 = V i 2
  1. Ускорение свободного падения: мальчик падает с ускорением на землю.Это ускорение называется ускорением свободного падения и обозначается g. числовое значение g составляет приблизительно 10 мс -2 у поверхности Земли.

CBSE Class 9 Science Примечания к главе: Движение

В этой статье представлены научные заметки CBSE Class 9 по главе 8 «Движение» (Часть II). В предыдущей части, то есть в научных заметках CBSE Class 9 по главе 8 «Движение» (Часть I), вы узнали о движении и его различных атрибутах.В этой части вы узнаете о трех уравнениях движения. Эти примечания к главе подготовлены экспертами в данной области и охватывают все важные темы главы. В конце заметок вы можете попробовать вопросы, заданные из обсуждаемого набора тем. Эти вопросы помогут вам отслеживать свой уровень подготовки и разобраться в предмете.

Также читайте: CBSE Class 9 Science notes по главе 8 «Движение» (Часть I)

Основные темы, затронутые в этой части CBSE Class 9 Science, Motion: Chapter Notes:

  • Три уравнения движения
  • Вывод трех уравнений движения

Простым математическим методом

Графическим методом

Также прочтите: Полный учебный материал по классу 9 CBSE на 2020-2021 годы

Ключевые примечания к главе — Фундаментальная единица жизни:

Уравнения равномерно ускоренного движения:

Есть три уравнения движения тел, движущихся с равномерным ускорением.Они объяснены ниже:

1. Первое уравнение движения:

(соотношение скорость-время)

v = u + at

Где, v = Конечная скорость тела

u = Начальная скорость тела

a = ускорение

И t = Время

Производство:

Использование формулы ускорения:

Программа CBSE по естествознанию, класс 9, 2020-2021 гг.

2.Второе уравнение движения:

(отношение положения и времени)

3. Третье уравнение движения:

Уравнения движения графическими методами:

1. Первое уравнение движения:

v = u + at

Рассмотрим следующий график скорости и времени:

2.Второе уравнение движения:

Пусть s будет пройденным расстоянием.

Как мы знаем, расстояние, пройденное объектом, определяется как площадь, ограниченная графиком,

(iii) Третье уравнение движения:

v 2 u 2 = 2 как

Рассмотрим график скорость-время:

Здесь расстояние, с = Площадь трапеции OABC

s = Площадь прямоугольника ADCO + Площадь треугольника ABD

Круговое движение
1.Равномерное круговое движение
: Когда объект движется по круговой траектории с постоянной или равномерной скоростью, говорят, что он имеет равномерное круговое движение. Например: движение стрелок часов, движение луны, вращающейся вокруг земли.
2. Неравномерное круговое движение: Когда объект движется по круговой траектории с переменной скоростью, говорят, что он имеет неравномерное круговое движение.
Когда объект совершает круговое движение, направление его скорости постоянно меняется.

Скорость объекта, движущегося по круговой траектории:

Предположим, что тело движется по круговой траектории с радиусом r .

Попробуйте ответить на следующие вопросы:

1 кв. Как определить расстояние до объекта по его графику скорость — время?

2 кв. Автомобиль трогается с места и разгоняется равномерно в течение 5,21 секунды на дистанции 110 м. Определите ускорение автомобиля.

3 кв. Изучите график скорости-времени

4 кв. Найдите следующее на основе графика

(i) Какой путь имеет постоянную скорость?

(ii) Когда достигается максимальная скорость?

(iii) По какому пути ускоряется движение?

(iv) По какому пути движение замедляется?

Q5. Автомобиль движется со скоростью 72 км / ч и применяет тормоза, обеспечивающие замедление 5 мс. -2 .

(i) Сколько времени требуется машине для остановки.

(ii) Какое расстояние проезжает машина, прежде чем остановиться?

(iii) Какой будет тормозной путь, если скорость автомобиля увеличится вдвое?

Также читайте: Примечания к главе по науке CBSE, класс 9 — все главы

Как три уравнения движения выводятся для класса 9

T h e 3 e q u a t i o s n f m o t i o n a r e : — 149 901 901 901 901 901 901 + a t

s = u t + 1 /

4 2 v ² u ² = 2 a s

W 9 e r e :

v — конечная скорость тела

50 — конечная скорость тела

s — расстояние / смещение тела

a — ускорение тела

t — затраченное время

N o t e : Три уравнения движения действительны только тогда, когда тело движется с u n i f o r м a c c e l 9211 3 e r a t i o n 13

50 .

Мы можем вывести 3 уравнения движения двумя способами.

• Графический метод

• Неграфический метод

Здесь мы выведем уравнения движения по N o n

19 r a p h i c a

50

50 50 e t h o d .

D e r i v a t i o n 9 12 9 12 9 12 9 12 9 12 9 12 9 12 9 12 912 9 t e q u a t i o n o f m

921 921 921 921 921 921 921

o n :

=> Ускорение = Изменение скорости / времени

=> a = vu / t

= > v = = = = = + a т (1 ) (1 ) 0

D e r i v a t i o n o 921 912 211112 921 912 21112 d e q u a t i o n o f м м o n :

Для равноускоренного движения

Vav = u + v / 2

Кроме того, смещение = (Средняя скорость) × (Время)

=> s = (u + v / 2) × t

=> s = (u + u + at / 2) t ( s i n c e , v = u + a t )

= 901 49> s = u t + 1 / 2 a t ² 1414 ( 2 )

D e r i v a 921 921 921 92 i113 12 n111 9211 921 92 o f 3 r d e q u a t 921 921 9 9 219 f m o t i o n :

Из (1), t = vu / a

Подстановка значение t во втором уравнении движения,

=> s = u (vu / a) + 1 / 2a (vu / a) ²

=> s = uv-u² / a + 1 / 2a ( v² + u²-2uv)

=> s = 2uv-2u² + v² + u²-2uv / 2a

=> s = v²-u² / 2a

= > v ² u ² = 2 a s

What is Kinematics — Get Kinematics Physics Notes, Books, Formulas, Equations

Кинематика — одна из самых основных глав в механике, и вы также найдете много применений кинематики в нашей повседневной жизни после прочтения этой статьи.Большую часть своей жизни мы ходили пешком, путешествуя отсюда туда, и поэтому мы также обеспокоены тем, сколько времени потребуется, чтобы переехать из одного места в другое. Кинематика занимается всеми этими процессами и упрощает нашу повседневную жизнь.

В Индии большинство из нас фанаты крикета, и если вы когда-нибудь замечали, когда боулер подает мяч, на табло также отображается скорость мяча, а иногда повторы также показывают возможную траекторию полета мяча, которую нужно определить для Leg Before Wicket ( LBW) тест.Это возможно, потому что кинематика помогает понять «движение» мяча. Движение — это не что иное, как смена позиции. В этой главе вы поймете некоторые основные термины, такие как расстояние, смещение, скорость, скорость, ускорение и т. Д. Вам следует прочитать эту главу очень внимательно, потому что вопросы из этой главы задаются не только на экзаменах 10 + 2, но и на различных других государственных экзаменах. в разделе «Общие способности».

Подготовка к работе в сети для JEE Main / NEET

Взломайте JEE 2021 с программой онлайн-подготовки JEE / NEET

Начать сейчас

Использование DRS для тестирования LBW

Кинематические темы

Система отсчета, Движение по прямой — скорость и скорость, Равномерное и неравномерное движение, средняя скорость и мгновенная скорость, равномерно ускоренное движение, положение-время, графики скорости-времени, отношения для равноускоренного движения.Скаляры и векторы, нулевой вектор, скалярные и векторные произведения, единичный вектор, скорость, разрешение вектора, сложение и вычитание векторов. Движение в плоскости, относительная скорость, движение снаряда, равномерное круговое движение.

Обзор кинематики

В кинематике нас не интересует причина движения, мы просто озабочены движением объекта, простыми словами, нас не волнует, сколько импульса мячу придает боулер, нас интересует только движение шара после того, как он потерял контакт с рукой котелка.Движение объекта может быть одномерным, двухмерным или даже трехмерным.

Движение объекта также может быть линейным или круговым. В этой главе вы познакомитесь с 3 уравнениями движения. Вы можете решить большинство вопросов кинематики, используя только эти три уравнения движения. Но прежде чем продолжить в этой главе, я предлагаю вам сначала развить некоторые базовые математические навыки, такие как дифференцирование, интегрирование, а также векторы. Я уверен, что вы изучали разницу между скалярными и векторными величинами в своих школьных учебниках.В этой главе вы познакомитесь с системой отсчета. Система отсчета — это система координат. Существует множество используемых систем координат, но наиболее распространена декартова система координат, состоящая из трех взаимно перпендикулярных осей. В действительности нет ничего стационарного, понятие стационарного используется в системе отсчета. Например, здание неподвижно, только если вы наблюдаете за ним на Земле, но если вы видите это здание с Луны, оно не неподвижно, поскольку Земля также вращается.Звучит интересно? Теперь вы можете разыграть своих одноклассников с помощью этой базовой системы координат.

Вначале вы также часто будете делать ошибки или вычитать величины, например, если мяч брошен вертикально вниз от здания со скоростью «u», то какой будет конечная скорость по прошествии времени t? Будет ли оно равно «u + gt» или «u-gt»? Поэтому, чтобы свести к минимуму подобную путаницу, вы всегда должны использовать вектор в векторных величинах. Вопросы из этой главы сложны, когда они представлены в графической форме.Вы должны ознакомиться с основными уравнениями линии, параболы, чтобы получить решение.

Предположим, ваш школьный автобус отправляется в 7:00, тогда какое расстояние он проедет до 7:45?

Раствор:

Дано

Формулы кинематики

Уравнения движения

Как подготовить кинематику?

Прежде чем решать вопросы, вы должны хорошо владеть основами исчисления, вы должны уметь различать и интегрировать функции.Чтобы хорошо усвоить концепции, вы должны сначала решить вопрос из одномерного движения, а затем решить только двухмерное движение. Также изучите основы построения графиков. После прочтения темы пройдите несколько пробных тестов по этой конкретной теме, чтобы набраться сил. Иногда вопросы, связанные с относительным движением, движением снаряда и проблемой реки, являются сложными, поэтому больше практикуйтесь по этим темам.

Примечания по кинематике

  • С помощью дифференцирования и интегрирования вы всегда можете найти функции x, v, a из одной заданной функции.Посмотрите на диаграмму ниже, чтобы понять это. Этот прием очень важен, потому что он будет использоваться во многих вопросах.

  • Самостоятельно составляйте графики для различных уравнений, с которыми вы сталкиваетесь при решении вопросов. Например. график x-t, график v-t, график a-t, график v-x, график a-x и т. д.

  • Используйте векторные обозначения в векторных величинах, направления очень важны, чтобы избежать глупых ошибок.

  • Не только запоминайте формулы, сначала проверьте их сами.

  • Для движения в 2-х или 3-х измерениях всегда разбивайте (разделяйте) движение на компоненты по осям.

  • Существует общая подсказка для всех типов вопросов, в то время как практические вопросы всегда пишут, какие количества указаны в вопросе и что вам нужно найти.

Книги по кинематике

В книгах NCERT вы найдете вопросы уровня от простого до умеренного, сначала ответьте на них.Затем вы можете ответить на вопросы из «Понимания физики» Д.К. Панди или Х.К. Верма. Обе эти книги содержат вопросы хорошего качества. Но помимо этих книг вам также следует проводить пробные онлайн-тесты по темам. Наша платформа предоставит вам видео по всем темам, статьи на простом языке, чтобы облегчить обучение, и множество вопросов в пробных тестах.

Примечания по физике для инженерных и медицинских экзаменов

Извините! — Страница не найдена

Пока мы разбираемся, возможно, вам поможет одна из ссылок ниже.

Дом
Назад

  • Класс
  • Онлайн-тесты
  • Ускоренный онлайн-курс JEE
  • Двухлетний курс для JEE 2021
  • Класс
  • Онлайн-курс NEET
  • Серия онлайн-тестов
  • CA Foundation
  • CA Средний
  • CA Финал
  • Программа CS
  • Класс
  • Серия испытаний
  • Книги и материалы
  • Тестовый зал
  • Умный взломщик BBA
  • Обучение в классе
  • Онлайн-коучинг
  • Серия испытаний
  • Взломщик Smart IPM
  • Книги и материалы
  • GD-PI
  • CBSE, класс 8
  • CBSE, класс 9
  • CBSE, класс 10
  • Класс 11 CBSE
  • CBSE, класс 12
  • Обучение в классе
  • Онлайн-классы CAT
  • Серия испытаний CAT
  • MBA Жилой
  • Умный взломщик CAT
  • Книги и материалы
  • Онлайн-классы без CAT
  • Серия испытаний без CAT
  • Тестовый зал
  • GD-PI
  • Обучение в классе
  • Серия испытаний
  • Интервью с Civils
  • Класс
  • Онлайн-классы
  • Серия испытаний SSC
  • Переписка
  • Практические тесты
  • Электронные книги SSC
  • Учебный пакет SSC JE
  • Класс
  • RBI, класс B
  • Банк испытаний серии
  • Переписка
  • Банковские электронные книги
  • Банк ПДП
  • Онлайн-коучинг
  • Обучение в классе
  • Серия испытаний
  • Книги и материалы
  • Класс
  • Программа моста GRE
  • GMAT Онлайн-коучинг
  • Консультации по приему
  • Консультации по GMAT
  • Стажировка
  • Корпоративные программы
  • Студенты колледжа
  • Работающие специалисты
  • Колледжи
  • школы

Уравнение движения графическим методом

Вопрос 1 Что подразумевается под равномерным круговым движением?

Вопрос 2 Тело движется вокруг Солнца с постоянной скоростью по круговой орбите.Движение равномерное или ускоренное?

Вопрос 3 Спутник движется вокруг Земли по круговой орбите с постоянной скоростью. Движение равномерное или ускоренное?

Вопрос 4 Приведите примеры кругового движения?

Вопрос 5 Что такое центростремительная сила. Приведите пример?

Вопрос 6 Велосипедист объезжает круговую дорожку каждые 5 минут. Если радиус круговой дорожки составляет 110 метров, рассчитать его скорость?

Вопрос 7 Вывести первое уравнение движения графическим методом?

Вопрос 8 Вывести второе уравнение движения графическим методом?

Вопрос 9 Вывести третье уравнение движения графическим методом?

Равномерное круговое движение

Когда тело движется по кругу, это называется круговым движением.
Когда тело движется по круговой траектории, направление его движения постоянно меняется.
Синусоидально изменяется скорость (из-за непрерывного изменения направления), поэтому движение по круговой траектории называется ускоренным.
Когда тело движется по круговой траектории с постоянной скоростью, его движение называется равномерным круговым движением .
Скорость тела, движущегося по кругу с постоянной скоростью, неодинакова, поскольку направление движения постоянно меняется.

Например:
Камень, привязанный к нити, вращается по круговой траектории с постоянной скоростью по часовой стрелке.
A ———> Скорость направлена ​​на восток.
B ———> Скорость направлена ​​на юг.
Поскольку есть изменение направления скорости, скорость не является равномерной.
Движение по кругу с постоянной скоростью является примером ускоренного движения.
Сила, необходимая для движения объекта по круговой траектории, называется центростремительной силой .

Например:
1) Движение искусственного спутника Земли.
2) Движение Луны вокруг Земли.
3) Движение Земли вокруг Солнца.
4) Наконечник секундной стрелки часов.
5) Спортсмен движется по круговой траектории.

Скорость тела, движущегося по круговой траектории, определяется по формуле:

в = 2нр / т

Уравнение движения графическим методом .

1) Вывод v = u + при
Начальная скорость u при A = OA
Скорость изменяется от A до B за время t (равномерное ускорение a)
Конечная скорость v = BC
BC = BD + DC
v = BD + AO
v = BD + u
Наклон графика зависимости скорости от времени равен ускорению a.
a = BD
a = BD / AD
a = BD / t
BD = при
v = u + при

2) Вывод S = ut +1/2 x при 2

Расстояние, пройденное телом, определяется площадью пространства между графиком скорости AB и временной осью OC, равным площади рисунка OABC.
Пройденное расстояние = Площадь рисунка OABC
= Площадь прямоугольника OADC + Площадь треугольника ABD
= (OA x OC) + 1/2 x AD x BD)
= (uxt) + (1/2 xtx at)
S. = ut + 1/2 x при 2

3) Вывод v 2 = u 2 + 2as

Расстояние, пройденное телом за время t, определяется площадью фигуры OABC (которая представляет собой трапецию)
с = Площадь трапеции OABC
с = Сумма параллельных сторон x высота / 2
с = (OA + OB) x OC / 2
s = (u + v) xt / 2
v = u + при
at = v — u
t = vu / a
s = (u + v) x (v- u) / 2a
2as = v 2 — u 2
v 2 = u 2 + 2as

Кинематика | физика | Britannica

Kinematics , раздел физики и подраздел классической механики, связанный с геометрически возможным движением тела или системы тел без учета задействованных сил ( i.е., причины и следствия движений).

Британская викторина

Викторина «Все о физике»

Кто был первым ученым, проведшим эксперимент по управляемой цепной ядерной реакции? Какая единица измерения для циклов в секунду? Проверьте свою физическую хватку с помощью этой викторины.

Далее следует краткое описание кинематики.Для полного обращения см. Механика .

Кинематика предназначена для описания пространственного положения тел или систем материальных частиц, скорости движения частиц (скорости) и скорости изменения их скорости (ускорения). Если пренебречь причинными силами, описание движения возможно только для частиц, имеющих ограниченное движение — , то есть , движущихся по определенным траекториям. В неограниченном или свободном движении силы определяют форму траектории.

Для частицы, движущейся по прямому пути, список положений и соответствующих моментов времени мог бы составить подходящую схему для описания движения частицы. Для непрерывного описания потребуется математическая формула, выражающая положение во времени.

Получите подписку Britannica Premium и получите доступ к эксклюзивному контенту.
Подпишитесь сейчас

Когда частица движется по криволинейной траектории, описание ее положения становится более сложным и требует двух или трех измерений.

Добавить комментарий

Ваш адрес email не будет опубликован. Обязательные поля помечены *